You are on page 1of 50

May

International
2017
1 1
Question-and-Answer Service

Reading Test
65 M I NU TES, 5 2 QUESTIONS

Turn to Section 1 of your answer sheet to answer the questions in this section.

Each passage or pair of passages below is followed by a number of questions. After reading
each passage or pair, choose the best answer to each question based on what is stated or
implied in the passage or passages and in any accompanying graphics (such as a table or
graph).

Questions 1-10 are based on the following old man’s listening ear, and to his wife’s ear, even
.......................................................................................................................................................................................................

passage. when the car was relatively far away and beyond their
This passage is adapted from Amit Chaudhuri, A Strange and
range of vision. They had pondered over the sound,
Sublime Address. ©1991 by Amit Chaudhuri. A ten-year-old 30 and finally, he had lit the lantern and shuffled out.
boy named Sandeep travels with his mother, his aunt “I told her,” he said, referring to his wife. “I told her
(Mamima), and his uncle (Chhotomama) to visit family in that I heard the car, I knew it was the car, I told her
Calcutta, India. you were coming.”
Once they were inside, Mamima gave the pot of
Two boys were playing carrom on the steps of a 35 yoghurt and the pot of sweetmeats to the old
small, painted shed which had the following words lady. “There was no need,” she said. “Oh really,” she
on its wall in large, black letters: NATIONAL said. “This is too much,” she insisted, with the air of
Line ASSOCIATION OF SPORTSMEN. A single one who has just received the Kohinoor diamond as
5 table-tennis table inside the shed could be glimpsed a birthday present. “Come, come, come,” said
through the window. The boys interrupted their 40 Chhotomama, with the air of someone who has just
game to give Chhotomama directions to the house given the Kohinoor diamond as a birthday present,
in a series of sporadic, enthusiastic gestures. Oh yes, and refuses to be overawed by his own generosity.
they knew the old couple. And yes, their son and “It’s nothing.” It was nothing, of course, only
10 daughter-in-law had arrived last night with their Ganguram’s sweets and yoghurt, but they fussed and
first child. 45 fussed and created the illusion that it was something,
“Is it a girl or a boy?” asked Mamima, rolling something unique and untasted and unencountered.
down the window. The son and the daughter-in-law emerged shyly
“A girl,” said the boy. from the anteroom. They both stooped gently to
15 Mamima rolled up her window before the touch Chhotomama’s feet, and Sandeep’s aunt’s and
mosquitoes came in. The two boys vanished behind 50 his mother’s feet, a traditional greeting and a mark of
them. When they reached the house, they found that obeisance towards one’s elders.
the old man was waiting on the verandah with a “Oh no no no,” said Chhotomama, struggling to
lantern in his hand. Moths were shuddering round keep the son’s hand away from his feet. “There’s no
20 and round the lantern, though the old man was need for all this.” This was half a token gesture
oblivious to them. He had come out because he had 55 towards modesty, and half towards the new,
heard the throbbing of the engine in the distance. “modern” India—Nehru’s secular India, free of ritual
The night had been silent except for the questioning and religion.
cry of an owl and the continual orchestral sound of
25 crickets in the bushes. The throbbing of the engine
had, therefore, travelled through the silence to the
April QAS 2017

Unauthorized copying or reuse of any part of this page is illegal. 2 CO NTI N U E


1 1
Question-and-Answer Service

“I have not met you for two years, Dada,” said the 3

...............................................................................................................................................................................................................................................................................................................
son, struggling to get his hands near Chhotomama’s
60 toes. “You must not stop me.” This was half a token As used in lines 37 and 40, “air” most nearly means
gesture towards modesty, and half towards the old, A) atmosphere.
“traditional” India—Gandhi’s India of ceremony and
custom. B) absence.
Sandeep, meanwhile, had come to the conclusion C) demeanor.
65 that the grown-ups were mad, each after his or her D) melody.
own fashion. Simple situations were turned into
complex, dramatic ones; not until then did everyone
feel important and happy. Will they never grow up? 4
thought Sandeep irately. He glanced around him. A
70 single blue, fluorescent tube was burning on the wall. The characters’ behavior during the gift giving
It was not a big room. Despite its bareness, the mainly serves to
impression it gave was of austerity rather than A) emphasize the lavish value of the gift.
poverty. It made one remember that poverty meant
displacement as well as lack, while austerity meant B) inflate the significance of the gesture.
75 being poor in a rooted way, within a tradition and C) convey indifference toward the gift.
culture of sparseness, which transformed even the D) stress the need for polite behavior.
lack, the paucity, into a kind of being.

5
1
Which choice provides the best evidence for the
According to the passage, the old man was standing answer to the previous question?
on the verandah because
A) Lines 43-44 (“It was . . . yoghurt”)
A) he was watching cars travel down the road.
B) Lines 44-46 (“they . . . unencountered”)
B) the two boys had reported the visitors would
soon arrive. C) Lines 52-54 (“Oh no . . . all this”)

C) he had heard what he believed to be the D) Lines 58-60 (“I have . . . stop me”)
visitors’ car.
D) he enjoyed listening to the quiet sounds of 6
the evening.
The description of Chhotomama and the son’s
interaction mainly serves to
2 A) show how the characters diverge in their
In the passage, the yoghurt and sweetmeats are approaches to cultural practices.
compared to a B) emphasize the characters’ complex relationship.
A) jewel. C) stress the characters’ misinterpretations of
B) cuisine. Indian history.

C) wedding gift. D) depict how the characters created gestures that


became routine.
D) generous donation.

April QAS 2017

Unauthorized copying or reuse of any part of this page is illegal. 3 CO NTI N U E


1 1
Question-and-Answer Service

7
Reading Test Questions 11-21 are based on the following

...............................................................................................................................................................................................................................................................................................................
passage and supplementary material.
Over the course of the passage, Sandeep comes to
view the adults as65 M I NU TES, 5 2 QUESTIONSThis passage is adapted from Nicholas Epley, Mindwise: How
We Understand What Others Think, Believe, Feel, and Want.
A) strict. ©2014 by Nicholas Epley.
Turn to Section 1 of your answer sheet to answer the questions in this section.
B) reserved. Knowing your own reputation can be surprisingly
C) sophisticated. difficult. Consider, for instance, a study that analyzed
a set of published experiments all sharing the same
D) immature.
Line basic design. In these experiments, people working in
5 a group would be asked to predict how the other
Each passage or pair of passages below is followed by a number of questions. After reading
group members would rate them on a series of
8 each passage or pair, choose the best answer to each question based on what is stated or
different traits. Researchers then compared these
implied in the passage or passages and in any accompanying graphics (such as a table or
Sandeep would be most critical of which action from predicted ratings to the other group members’ actual
the passage? graph). ratings on the very same traits. The traits varied from
10 one experiment to another and included qualities like
A) The two boys playing carrom intelligence, sense of humor, consideration,
B) Mamima’s inquiry about the gender of the child defensiveness, friendliness, and leadership ability.
Questions 1-10 are based on the following old
Theman’s
groupslistening ear, and to hiswithwife’s theear, even
.......................................................................................................................................................................................................

C) The old lady’s reaction to the gift varied in familiarity, members


passage. when
of some groups being fairly unfamiliar with one their
the car was relatively far away and beyond
D) The son and daughter-in-law waiting in
This passage is adapted from Amit Chaudhuri, A Strange and
range of vision. They had pondered over the sound,
15 another (such as having met only once, in a job
the anteroom
Sublime Address. ©1991 by Amit Chaudhuri. A ten-year-old 30 and finally,
interview) and he had lit the lantern
the members and groups
of other shuffledbeingout.
boy named Sandeep travels with his mother, his aunt “I told her,” he said, referring to his
very familiar with one another (such as having lived wife. “I told her
(Mamima), and his uncle (Chhotomama) to visit family in that I heard the car, I knew it was
together for an extended time as roommates). If the car, I told her
9 Calcutta, India. you
peoplewere knewcoming.”
exactly what others were thinking, then
Once
there would they be were inside,correspondence
a perfect Mamima gave between the pot of
Twolines
Which boysfrom
weretheplaying carrom
passage moston the steps
strongly of a
suggest 20
35 yoghurt and the pot of sweetmeats to the old
small,
that painted
India shed which had
has experienced social thechange?
following words predicted and actual ratings. If people were clueless,
lady. “There
then there was no
would be need,” she said. “Ohbetween
no correspondence really,” shethe
on its wall in large, black letters: NATIONAL said. “This is too much,” she insisted, with the air of
A) Lines 36-37 (“There was
Line ASSOCIATION OF SPORTSMEN. A single
. . . she insisted”) two. Statistically speaking, you measure relationships
one who has
like these withjust received thewhere
a correlation, Kohinoorperfectdiamond as
5 B) Lines 48-51
table-tennis table(“They
inside both
the shed. . . elders”)
could be glimpsed a birthday present. “Come, come,
25 correspondence yields a correlation of 1 and no
come,” said
through
C) Linesthe window.
54-57 (“ThisThe
wasboys interrupted their
. . . religion”) 40 Chhotomama, with the air of someone who has just
game to give Chhotomama directions to the house correspondence yields a correlation of 0. The closer
D) Lines 73-76 (“It made . . . sparseness”) given the Kohinoor
the correlation is to diamond as a birthday
1, the stronger present,
the relationship.
in a series of sporadic, enthusiastic gestures. Oh yes, andFirst,
refuses to be overawed by his own generosity.
they knew the old couple. And yes, their son and the good news. These experiments suggested
“It’s
that nothing.”
people was nothing,
areItpretty of course,
good, overall, only how
at guessing
10 daughter-in-law had arrived last night with their
10 first child. Ganguram’s sweets and yoghurt, but
30 a group of others would evaluate them, on average.
they fussed and
45 fussed and created the illusion that it was something,
As “Is it in
used a girl
lineor72,
a boy?” asked Mamima,
“impression” rolling
most nearly means The overall correlation in these experiments between
something unique and and
predicted impressions untasted and unencountered.
the average actual
down the window. The son and the daughter-in-law emerged
A) “Aappearance.
girl,” said the boy. impression of the group was quite high (.55, ifshyly
you
from
are the anteroom.
quantitatively They
inclined).bothTo stooped
put that gently
in to
15 B) Mamima
belief. rolled up her window before the touch Chhotomama’s feet, and Sandeep’s aunt’s and
35 perspective, this is roughly the same magnitude as
mosquitoes
C) imitation. came in. The two boys vanished behind 50 his mother’s feet, a traditional greeting and a mark of
them. When they reached the house, they found that the correlation between the heights of fathers and the
D) recollection. obeisance
heights of towards
sons (aroundone’s .5).
elders.
It is not perfect insight,
the old man was waiting on the verandah with a “Oh no no no,” said Chhotomama, struggling
lantern in his hand. Moths were shuddering round but it is also very far from being clueless. In otherto
keep
words, you probably have a decent sense of what no
the son’s hand away from his feet. “There’s
20 and round the lantern, though the old man was
need for all this.” This was half a token gesture
40 others generally think of you, on average.
oblivious to them. He had come out because he had 55 towards modesty, and half towards the new,
heard the throbbing of the engine in the distance. Now the bad news. These experiments also
“modern”
assessed India—Nehru’s
how well people couldsecular India,the
predict free of ritual
The night had been silent except for the questioning and religion.
cry of an owl and the continual orchestral sound of impression of any single individual within a given
25 crickets in the bushes. The throbbing of the engine group. You may know, for instance, that your
had, therefore, travelled through the silence to the
April QAS 2017

Unauthorized copying or reuse of any part of this page is illegal. 4


2 CO NTI N U E
1 1
Question-and-Answer Service

45 coworkers
“I have notin general
met you think
for two you years,
are ratherDada,” smart,
saidbut
the 3 Mean Correlations of Perceptions of

...............................................................................................................................................................................................................................................................................................................
those
son, struggling
coworkerstoalso get vary
his handsin their
nearimpression
Chhotomama’s of you. Individuals among New Acquaintances and
60 Some
toes. “You
thinkmustyou are
not asstop
sharpme.”asThis
a knife.
wasOthers
half a token
think As used in lines 37 and 40, “air” most nearly means
Old Acquaintances in Twenty-One Studies
you
gesture
are towards
as sharp modesty,
as a spoon. and Dohalf
youtowards
know the the old, A) atmosphere.
difference?
“traditional” India—Gandhi’s India of ceremony and
B) absence.1.0

0 = complete disagreement)
50 custom.
Evidently, no. The accuracy rate across these

(1 = complete agreement;
0.9 new acquaintance
experiments
Sandeep, meanwhile,
was barely better had come than to random
the conclusion
guessing C) demeanor.
0.8

Mean correlations
65 (an
thatoverall
the grown-ups
correlation wereof mad,
.13 between
each after predicted
his or her and well acquainted
D) melody.0.7
actual
own fashion.
evaluations,
Simple only
situations
slightlywerehigher turned
than into
no 0.6
relationship
complex, dramaticwhatsoever).
ones; not Although
until then youdidmighteveryone
have 0.5
55 some
feel important
sense of how and happy.
smart your Willcoworkers
they neverthink grow you up? 0.4
4 0.3
are,
thought
you Sandeep
appear toirately.
have no Heclueglanced
aboutaroundwhich him. A
0.2 behavior during the gift giving
The characters’
70 coworkers
single blue,influorescent
particular tubefind wasyou burning
smart and onwhich
the wall.
do 0.1to
mainly serves
not.
It wasAsnot
onea author
big room. of the
Despite
studyits writes,
bareness,“Peoplethe seem 0
to
impression
have just ita tiny
gaveglimmer
was of austerity
of insightrather
into than
how they A value ofBthe gift. C
A) emphasize the lavish
60 are
poverty.
uniquely
It made
viewedoneby remember
particularthat otherpoverty
people.”
meant
displacement
But perhaps asthis
wellisasholding
lack, whileyourausterity
mind-reading meant B) Ainflate
= correlation between individuals’
the significance self-perception
of the gesture.
and those individuals’ predictions of how others
75 abilities
being poor to too
in ahigh
rooteda standard?
way, within It’s ahard,
tradition
after and
all, to C) perceive
convey them
indifference toward the gift.
define
culturetraits
of sparseness,
like intelligence
which transformed
and trustworthiness even the D) stress the need for polite behavior.
precisely,
lack, the paucity,
so it might intonot
a kindbe soofsurprising
being. that we B = correlation between individuals’ self-perception
65 have difficulty guessing how others will evaluate us
and actual perception of those individuals by others
on these ambiguous traits. What about predicting 5
1 something simpler, such as how much other people C = correlation between individuals’ predictions of
like you? Surely you are better at this. You learn over Which choice provides the best evidence for the
According toaround
the passage, thewhoold smile
man was standing how others perceive them and actual perception of
time to hang people at you and answer to the previous question?
on the verandah because those individuals by others
70 avoid those who spit at you. You must have a much
A) Lines 43-44 (“It was . . . yoghurt”)
better
A) hesense of who likes
was watching carsyou anddown
travel who the hates you
road.
within a group. Yes? B) Lines
Adapted from44-46
Erika N.(“they
Carlson. and
. . unencountered”)
Simine Vazire, “Meta-Insight:
B) I’m
theafraid
two boysnot.hadThesereported
studiesthe visitors
found thatwould
people are Do People Really Know How Others See Them?” ©2011
soon arrive. C) Lines 52-54 (“Oh no . . . all this”)
by American Psychological Association.
only slightly better than chance at guessing who in a
C) helikeshad heard whatwhohe believed D) Lines 58-60 (“I have . . . stop me”)
75 group them and does notto (thebe average
the
visitors’here
correlation car. was a meager .18). Some of your
coworkers
D) he enjoyed like you and others
listening to thedo not,sounds
quiet but I wouldn’t
of 6
11
count on you
the evening. knowing the difference. The same
barely-better-than-guessing accuracy is also found in The
Which description of Chhotomama
choice best and the
supports the claim son’s
in the
80 experiments investigating how well speed daters can interaction mainly
first sentence of theserves to
passage?
2 assess who wants to date them and who does not, A) show
Lines how the characters
2-4 (“Consider diverge in their
. . . design”)
how well job candidates can judge which
In the passage,
interviewers theimpressed
were yoghurt and by sweetmeats
them and which are were approaches to cultural practices.
B) Lines 21-23 (“If people . . . two”)
compared to a B) emphasize
not, and even how well teachers can predict their C) Lines 26-27the characters’
(“The closer . .complex relationship.
. relationship”)
85 course evaluations. Granted, it’s rare that you are C) stress the characters’ misinterpretations of
A) jewel. D) Lines 54-58 (“Although . . . not”)
completely clueless about how you are evaluated. Indian history.
B) cuisine.
Accuracy tends to be better than chance in these
experiments,
C) wedding gift. but not necessarily by very much. D) depict how the characters created gestures that
became routine.
D) generous donation.

April QAS 2017

Unauthorized copying or reuse of any part of this page is illegal. 5


3 CO NTI N U E
1 1
Question-and-Answer Service

12
Reading Test 15

...............................................................................................................................................................................................................................................................................................................
The information about statistical measurement in As used in line 35, “magnitude” most nearly means
65 M.I.NU
lines 23-27 (“Statistically TES, 5 2
. relationship”) is QUESTIONSA) strength.
presented in order to
B) influence.
Turn to
A) correct a common Section 1 of yourofanswer
misunderstanding how sheet to answer the questions in this section.
researchers quantify certain data from C) severity.
experiments. D) reality.
B) forestall potential objections to how data from
the experiments were analyzed in the study.
Each passage or pair of passages below is followed 16 by a number of questions. After reading
C) draw attention to apassage
each patternor evident in the the best answer to each question based on what is stated or
pair, choose
conclusions of the experiments. What main effect do the words “clueless” (line 38)
implied in the passage or passages and in any accompanying graphics (such as a table or
and “mind-reading” (line 61) have on the tone of the
D) provide context for a way in which the results of
graph). passage?
the experiments will be discussed.
A) They contribute to a casual and gently humorous
tone that renders a potentially specialized
13 Questions 1-10 are based on the following old man’s listening
discussion moreear, and to his wife’s ear, even
approachable.
.......................................................................................................................................................................................................

passage. when the car was relatively


B) They contribute to a slyly far mocking
away andand beyond their
Based on the passage, in which situation would an
This passagestand
rangedisapproving tone that reinforces thethe
of vision. They had pondered over sound,
author’s
individual the greatest
is adapted from Amit chance of accurately
Chaudhuri, A Strange and
Sublime Address. 30 and criticisms of the researchers’ conclusions. out.
finally, he had lit the lantern and shuffled
predicting how©1991
he or by
sheAmit Chaudhuri. A ten-year-old
is perceived?
boy named Sandeep travels with his mother, his aunt “I told her,” he said, referring to his wife. “I told her
A) An intern
(Mamima), predicts
and his the impression
uncle (Chhotomama) thatfamily
to visit her direct
in
C)
that They
I heardcontribute
the car, Ito a deeply
knew it waspessimistic toneher
the car, I told that
supervisor
Calcutta, India. holds of her. stresses
you were coming.”the impossibility of ever knowing how
people
Once they truly
were perceive
inside,each
Mamimaother.gave the pot of
B) Two
A manager
boys were predicts
playing thecarrom
collective opinion
on the steps of
of a
employees about her ability. yoghurt
35 D) Theyand the pot of
contribute tosweetmeats
a thoughtfultoyet theuncertain
old
small, painted shed which had the following words lady.tone
“There
thatwascastsnodoubt
need,”onshethesaid. “Oh really,” she
real-world
on its
C) Anwall in large,predicts
instructor black letters: NATIONAL
the enthusiasm of his said.usefulness
“This is too ofmuch,” she insisted,
experimental data. with the air of
Line ASSOCIATION OF SPORTSMEN.
class after talking with two students. A single one who has just received the Kohinoor diamond as
5 table-tennis table inside the shed could be glimpsed
D) A biographer predicts the esteem in which he is a birthday present. “Come, come, come,” said
through the window. The boys interrupted their 40 Chhotomama, with the air of someone who has just
held by the living subject of his book. 17
game to give Chhotomama directions to the house given the Kohinoor diamond as a birthday present,
in a series of sporadic, enthusiastic gestures. Oh yes, The author quoted
and refuses in lines 58-60
to be overawed by hisexpresses which
own generosity.
they knew the old couple. And yes, their son and view of the study’s results?
“It’s nothing.” It was nothing, of course, only
14
10 daughter-in-law had arrived last night with their
Ganguram’s
A) They indicatesweetsthatandthere
yoghurt, but they
is a small but fussed and
promising
first child.
Which choice provides the best evidence for the 45 fussed and created the illusion thathow
it was something,
“Is itto
answer a girl or a boy?”question?
the previous asked Mamima, rolling chance of correctly predicting one is
something
perceived.unique and untasted and unencountered.
down the window. The son and the daughter-in-law emerged shyly
A) “ALines 9-13 (“The
girl,” said the boy. traits . . . familiarity”) B) They show that individuals generally know very
from the anteroom. They both stooped gently to
15 B) Lines 28-33 (“These experiments
Mamima rolled up her window before the . . . high”) little about how they are regarded by groups of
touch Chhotomama’s feet, and Sandeep’s aunt’s and
mosquitoes
C) Lines 41-44 came(“Now
in. The thetwo. . .boys vanished behind
group”) people.
50 his mother’s feet, a traditional greeting and a mark of
them. When they reached the house, they found that C) They reveal
obeisance towards that oneelders.
one’s individual hardly knows
D) Lines
the old man 68-70
was (“Surely
waiting on . . . the
at you”)
verandah with a what another individual
“Oh no no no,” said Chhotomama, thinks of him or her.to
struggling
lantern in his hand. Moths were shuddering round keepThey
the son’s handthataway from his feet. “There’s no
20 and round the lantern, though the old man was
D) confirm one’s predictions about other
needpeople’s
for all this.” This wasare
impressions halfno abetter
token than
gesture
random
oblivious to them. He had come out because he had 55 towards modesty, and half towards the new,
heard the throbbing of the engine in the distance. guesses.
“modern” India—Nehru’s secular India, free of ritual
The night had been silent except for the questioning and religion.
cry of an owl and the continual orchestral sound of
25 crickets in the bushes. The throbbing of the engine
had, therefore, travelled through the silence to the
April QAS 2017

Unauthorized copying or reuse of any part of this page is illegal. 6


2 CO NTI N U E
1 1
Question-and-Answer Service

18 “I have not met you for two years, Dada,” said the 3
20

...............................................................................................................................................................................................................................................................................................................
son, struggling to get his hands near Chhotomama’s
The main reason that the author includes the
60 toes. “You must not stop me.” This was half a token As usedstatement
Which in lines 37best
andexemplifies
40, “air” most
the nearly means
distinction
information
gesture towards about speed daters,
modesty, jobtowards
and half candidates, and
the old, made by correlation C in the figure?
teachers in lines 78-85 is to A) atmosphere.
“traditional” India—Gandhi’s India of ceremony and A)
custom. B) Sally believes she is outgoing but thinks that
absence.
A) caution against making assumptions about others will describe her as reserved.
Sandeep, meanwhile,motives.
certain individuals’ had come to the conclusion C) demeanor.
65 that the grown-ups were mad, each after his or her
B) Sally expects that others will say she is outgoing,
B) D) but
melody.
owndistinguish among
fashion. Simple certain behaviors
situations were turnedobserved
into in many describe her as reserved.
three different scenarios.
complex, dramatic ones; not until then did everyone C) Sally has been told that she is outgoing but only
C) indicate certain
feel important settings
and happy. where
Will theyfurther studyup?
never grow by 4 by people with whom she is well acquainted.
researchers
thought Sandeepisirately.
needed.He glanced around him. A D) Sally is outgoing with those with whom she is
single blue, fluorescent tube was in burning The characters’ behavior during the gift giving
70 D) offer examples of situations which on the wall.
a certain well acquainted but reserved around new
It was not a big room. Despite its bareness, the mainly serves to
finding holds true. acquaintances.
impression it gave was of austerity rather than A) emphasize the lavish value of the gift.
poverty. It made one remember that poverty meant
B) inflate the significance of the gesture.
19 displacement as well as lack, while austerity meant 21
75 being poor in a rooted way, within a tradition and C) convey indifference toward the gift.
According to the figure,
culture of sparseness, the mean
which correlation
transformed eventhat
the Information in the figure is most useful for
most nearly approaches complete agreement exists D) stress the
addressing needquestion
which for polite behavior.by the passage?
provoked
lack, the paucity, into a kind of being.
between individuals’ self-perception and
A) What determined the traits that researchers
A) how those individuals are actually perceived by 5 tended to focus on in the experiments being
1 new acquaintances. analyzed?
Which choice provides the best evidence for the
According
B) to the passage,
actual perceptions the old
of those man was standing
individuals by people B) Why
answer toare
theindividuals more likely to accurately
previous question?
on the verandah
with because
whom they are well acquainted. predict the impressions of groups than of
A) specific
Lines 43-44 (“It waswithin
individuals . . . yoghurt”)
groups?
C)
A) the individuals’
he was watchingpredictions of howthe
cars travel down they are
road.
perceived by people with whom they are well B) To
C) Lines 44-46
what (“they
degree are. people
. . unencountered”)
able to predict how
B) the two boys had reported the visitors would
acquainted. C) individual
Lines 52-54acquaintances
(“Oh no . . . all perceive
this”) them?
soon arrive.
D) the predictions those individuals make about D) Is one58-60
Lines person’s understanding
(“I have of trustworthiness
. . . stop me”)
C) he had heard what he believed to be the
how they are perceived by new as well as old really so different from another person’s
visitors’ car.
acquaintances. understanding of that trait?
D) he enjoyed listening to the quiet sounds of 6
the evening.
The description of Chhotomama and the son’s
interaction mainly serves to
2 A) show how the characters diverge in their
In the passage, the yoghurt and sweetmeats are approaches to cultural practices.
compared to a B) emphasize the characters’ complex relationship.
A) jewel. C) stress the characters’ misinterpretations of
B) cuisine. Indian history.

C) wedding gift. D) depict how the characters created gestures that


became routine.
D) generous donation.

April QAS 2017

Unauthorized copying or reuse of any part of this page is illegal. 7


3 CO NTI N U E
1 1
Question-and-Answer Service

Reading Test
Questions 22-32 are based on the following carbon-rich gunk—just the sort of residue that might

...............................................................................................................................................................................................................................................................................................................
passage and supplementary material. have been left behind on an icy object that had its
outermost layers vaporised in the bright sunlight of
This passage is adapted 65 from M IDavid
NUShiga,TES, “Has5Pluto
2 QUESTIONS
Sent
the inner solar system. Bottke and Levison’s
Us a Message in Ceres?” ©2008 by Reed Business
Information, Ltd. 50 computer simulations show that the observed
Turn to Section 1 of your answer sheet to answer number of objectsinisthis
the questions about right if they are
section.
Does Pluto have a wayward cousin lurking in the immigrants, though they have assumed many of the
inner solar system? The dwarf planet Ceres—and objects broke up after transport.
other icy chunks—may have been born in the same Thomas McCord of the Bear Fight Center in
Line realm as Pluto, but travelled all the way to the 55 Winthrop, Washington, who was not involved in any
5 asteroid belt between Eachthe orbitsorofpair
passage Mars of and Jupiter.
passages below is followedofbythe three studies,
a number agrees that
of questions. Afterthe asteroid belt
reading
If so, it would be further evidence that a massive
each passage or pair, choose the best answer to each question based on what is statedfrom
probably hosts some small refugees or the outer
upheaval rearranged the early solar system.
implied in the passage or passages and in any accompanying graphics (such as a table or to believe
solar system, but says there is no reason
At 950 kilometres in diameter, Ceres is by far the Ceres is a stranger there. Its ice-to-rock ratio matches
largest object in the graph).
asteroid belt. And that’s not the 60 the expected composition of the raw materials that
10 only reason it doesn’t quite fit in with many of its would have been available at its current position
companions, according to William McKinnon of early on, he says. What’s more, objects of its size are
Washington University. expected
Questions 1-10 are based on the following old man’stolistening
have formed ear, andin the inner
to his solar
wife’s ear,system.
even
.......................................................................................................................................................................................................

McKinnon points out that Ceres has a low New measurements of Ceres’s composition by their
passage. when the car was relatively far away and beyond
density, which suggests it is 25 to 30 per cent water 65 NASA’s
15 ice. That’s
This passageaishigh proportion
adapted from Amit forChaudhuri,
an asteroid, but and
A Strange
range of Dawn
vision.mission,
They had forpondered
which McCord
over theissound,
a team
member, could help pin down its birthplace.
30 and finally, he had lit the lantern and shuffled out.
Sublime Address. ©1991
closely matches Plutoby andAmit Chaudhuri.
other A ten-year-old
icy objects native to
boy namedsolar
the outer Sandeep travels
system, with his
known as mother, his aunt
trans-Neptunian “I told her,” heofsaid,
Properties referring
Selected SolartoSystem
his wife. “I told her
Objects
(Mamima), and his uncle (Chhotomama)
objects (TNOs). What’s more, a dip in Ceres’s light to visit family in that I heard the car, I knew it was the car, I told her
Average
Calcutta, India.
spectrum may be a sign of ammonium-rich clay at you were coming.”
distance Average
theTwo
surface. Once they were inside,Radius Mamima gave Massthe pot of
20 boysThiswerematerial
playinghas neveron
carrom been
the found
steps ofina Object from Sun
35 yoghurt and the pot of(Earth sweetmeats density
the fragments = 1) to the old
(Earth = 1)
small, painted of asteroids
shed which that
had thehavefollowing
fallen to words
Earth, (Earth
lady. “There was no need,” she said. “Oh really,” she ) (g/cm 3

but fits the expected ammonia-rich


on its wall in large, black letters: NATIONAL composition distance
of a TNO. said. “This is too =much,”
1) she insisted, with the air of
Line ASSOCIATION OF SPORTSMEN. A single Earth
one who has 1.0
just received 1.0Kohinoor 1.0
the diamond as 5.5
5 So if Ceres
table-tennis formed
table inside in the
Pluto’s
shedneighbourhood,
could be glimpsed Mars 1.52 0.53 0.11 3.9
how did the
it end up 2 toThe 4 billion kilometres away? a birthday present. “Come, come, come,” said
25 through window. boys interrupted their Juno
40 Chhotomama, with the air of someone who has just
Some
game to researchers think thatdirections
give Chhotomama the orbits to of the
the house
planets 2.7 0.019 0.000003 2.8
(asteroid)
given the Kohinoor diamond as a birthday present,
were once unstable. According to this
in a series of sporadic, enthusiastic gestures. Oh yes, idea—known
as theknew
Nicethe model—Uranus andyes,
Neptune went and refuses to be overawed by his own generosity.
Ceres
they old couple. And their son and “It’s nothing.”2.8 It was nothing, of course, only
rampaging through (dwarf 0.073 0.00015 2.7
10 daughter-in-law hadthe outerlast
arrived solar system
night witharound
their Ganguram’s sweets and yoghurt, but they fussed and
3.9 planet)
30 firstbillion
child. years ago. As a result, many of the icy 45 fussed and created
objects
“Is itthat
a girlformed in theasked
or a boy?” outerMamima,
solar system rollingwere Jupiter 5.2 the illusion 11.2 that it was318.0something, 1.3
pulled inward by the gravity of the two planets, and something
Saturn 1 unique
9.5 and untasted
9.5 and unencountered.
95.0 0.7
down the window.
some“Aended up joining
girl,” said the boy.the rocky asteroids that were UranusThe son and 19.2the daughter-in-law
4.0 emerged
15.0 shyly1.3
born in the asteroid belt. from the anteroom. They both stooped gently to
15 Mamima rolled up herCeres
window would simply
before the be the Neptune
touch
30.1
Chhotomama’s feet,
3.9
and Sandeep’s
17.0
aunt’s and
1.6
35 largest of these immigrants. “The
mosquitoes came in. The two boys vanished behind odds for this seem Pluto
50 his mother’s feet, a traditional greeting and a mark of
low,
them.but Whenit is not
theyinconceivable,”
reached the house, says they
Bill Bottke
found of that (dwarf 39.5 one’s elders. 0.2 0.002 2.1
the Southwest Research obeisance towards
old man was waitingInstitute (SwRI) inwith
on the verandah Boulder,
a planet)“Oh no no no,” said Chhotomama, struggling to
Colorado.
lantern in his hand. Moths were shuddering round keep the son’s hand away from his feet. “There’s no
20 andBottke
roundand the Hal Levison
lantern, of SwRI
though the oldledman
a pair wasof
need
1
for all
Properties giventhis.” This
do not wasSaturn’s
include half a ring
token gesture
system.
40 studies
oblivious which support
to them. He hadthe idea
comeofout refugees
because from the
he had 55 towards
Source: modesty,
Data from the and
Nationalhalf towards
Aeronautics the
and new,
Space
outer
heard solar system orbiting
the throbbing in the asteroid
of the engine belt.
in the distance.
They focused on the so-called D- and P-type “modern”
Administration India—Nehru’s
(NASA). secular India, free of ritual
The night had been silent except for the questioning and religion.
asteroids
cry of an owl thatandcomprise 20 per cent
the continual of the population
orchestral sound of
25 in the outer part of the belt. These
crickets in the bushes. The throbbing of the objects areengine
a dark
45 reddish colour that suggests they are
had, therefore, travelled through the silence to the covered in

April QAS 2017

Unauthorized copying or reuse of any part of this page is illegal. 8


2 CO NTI N U E
1 1
Question-and-Answer Service

22 “I have not met you for two years, Dada,” said the 3
25

...............................................................................................................................................................................................................................................................................................................
son, struggling to get his hands near Chhotomama’s
In McKinnon’s view, Ceres differs from other objects
60 toes. “You must not stop me.” This was half a token lines34,
As used in line 37 “simply”
and 40, “air”
mostmost nearly
nearly means
means
in the asteroid
gesture towardsbelt in which
modesty, significant
and half towardsway?the old,
A) wholly.
atmosphere.
“traditional”
A) The surfaceIndia—Gandhi’s
temperature ofIndia
Ceresofisceremony
lower than and
custom. B) sincerely.
absence.
the temperatures of the other objects.
Sandeep, meanwhile, had come to the conclusion C) plainly.
demeanor.
B) The dimensions of Ceres have varied more over
65 that the grown-ups were mad, each after his or her D) merely.
melody.
owntime thanSimple
fashion. the dimensions
situationsofwere
the turned
other into
objects have.
complex, dramatic ones; not until then did everyone
C) The surfaceand
feel important composition
happy. Willofthey
Ceres is dissimilar
never grow up?to 26
4
the composition
thought of the
Sandeep irately. Heother
glancedobjects.
around him. A
single
70 D) blue, fluorescent tube was isburning on the than
wall. Based on the passage,
The characters’ Bottke
behavior andthe
during Levison’s
gift giving
The light reflected by Ceres more intense
It was not a big room. Despite its bareness, the mainly serveswould
conclusions to be most weakened by a study that
the light reflected by the other objects.
impression it gave was of austerity rather than A) confirmed
emphasize that heat from
the lavish valuethe Sungift.
of the burned away
poverty. It made one remember that poverty meant the outer layers of all immigrant objects.
B) inflate the significance of the gesture.
23 displacement as well as lack, while austerity meant B)
75 being poor in a rooted way, within a tradition and C) established that the orbits
convey indifference towardofthe
certain
gift. objects of
Which
culture choice provideswhich
of sparseness, the best evidence for
transformed eventhethe the inner solar system were once less stable.
answer to the previous question? D) stress the need for polite behavior.
lack, the paucity, into a kind of being. C) demonstrated that very few objects broke up
A) Lines 2-5 (“The dwarf . . . Jupiter”) after migrating to the asteroid belt.
B) Lines 8-9 (“At 950 . . . belt”) 5 D) proved that not all immigrants from the outer
1 solar system survive in the asteroid belt today.
C) Lines 18-20 (“What’s . . . surface”) Which choice provides the best evidence for the
According to the passage, the old man was standing answer to the previous question?
D) Lines
on the 20-23 (“This
verandah because. . . TNO”)
27 A) Lines 43-44 (“It was . . . yoghurt”)
A) he was watching cars travel down the road.
B) Lines
Which 44-46
choice (“they .the
provides . . unencountered”)
best evidence for the
24 B) the two boys had reported the visitors would
answer to 52-54
C) Lines the previous
(“Oh noquestion?
. . . all this”)
soon arrive.
According to the passage, the Nice model is based on
C) idea
the thatheard what he believed to be the
he had A)
D) Lines 39-41
58-60 (“Bottke
(“I have ......stop
belt”)
me”)
visitors’ car. B) Lines 42-44 (“They . . . belt”)
A) Uranus and Neptune were not always locked
D) into
he enjoyed listening
their current to thepaths.
orbital quiet sounds of 6 C) Lines 44-49 (“These . . . system”)
the evening. D) Lines 49-53 (“Bottke . . . transport”)
B) Ceres traveled a greater distance than any other The description of Chhotomama and the son’s
object in the solar system did. interaction mainly serves to
2 C) objects formed in the inner solar system were 28 A) show how the characters diverge in their
able to resist the gravitational pull of Uranus and
In the passage, the yoghurt and sweetmeats are approaches
As used to cultural
in line 60, practices.
“raw” most nearly means
Neptune.
compared to a B) emphasize the characters’ complex relationship.
D) icy objects like Ceres were formed in the inner A) original.
A) solar
jewel.system. C) stress the characters’ misinterpretations of
B) young. history.
Indian
B) cuisine.
C)
D) exposed.
depict how the characters created gestures that
C) wedding gift.
D) became
inexperienced.
routine.
D) generous donation.

April QAS 2017

Unauthorized copying or reuse of any part of this page is illegal. 9


3 CO NTI N U E
1 1
Question-and-Answer Service

29
Reading Test Questions 33-42 are based on the following

...............................................................................................................................................................................................................................................................................................................
passages.
The last sentence of the passage serves mainly to
65 M I NU TES, 5 2 QUESTIONSPassage 1, by Patrick Henry, and Passage 2, by Edmund
A) hint at a possible weakness in a claim. Pendleton, are adapted from speeches delivered to the
B) emphasize the critical nature of a decision. Virginia ratifying convention in 1788. Both are in response
Turn to Section 1 of your answer sheet to answer
to the questions
the proposal in this
by the 1787section.
Constitutional Convention in
C) allude to a potential resolution to a puzzle. Philadelphia to replace the Articles of Confederation with a
D) reconcile two opposing positions on an issue. new constitution establishing a national government.
Passage 1
If a wrong step be now made, the republic may be
30 Each passage or pair of passages below is followedlost
by forever.
a numberIf of questions.
this After reading
new government will not come up
each passage or pair, choose the best answer to each
to question
the basedof
expectation onthewhat is stated
people, and or
they shall be
According to the table, which object has the lowest
implied in the passage or passages and in anyLine
accompanying graphics
disappointed, their (such as
liberty a table
will be or and tyranny
lost,
average density?
graph). 5 must and will arise.
A) Earth . . . And here I would make this inquiry of those
B) Ceres worthy characters who composed a part of the late
federal Convention. I am sure they were fully
C) Saturn 1-10 are based on the following
Questions old man’s listening
impressed with the ear, and toofhis
necessity wife’s ear,
forming a greateven
.......................................................................................................................................................................................................

passage.
D) Pluto 10 when the car government,
consolidated was relativelyinsteadfar away ofand beyond their
a confederation.
This passage is adapted from Amit Chaudhuri, A Strange and
range
That thisof vision. They had pondered
is a consolidated government overis the sound,
Sublime Address. ©1991 by Amit Chaudhuri. A ten-year-old 30 and finally, he clear;
demonstrably had litandthethe
lantern
danger andofshuffled
such a out.
31 boy named Sandeep travels with his mother, his aunt “I told her,” he
government is, said,
to myreferring
mind, very to his wife. “I told her
striking.
(Mamima), and his uncle (Chhotomama) to visit family in thatI have
I heard thethe car, I veneration
highest knew it wasfor thethose
car, Igentlemen;
told her
Which statement is supported by data represented in you
Calcutta, India. 15 but, were
sir, givecoming.”
me leave to demand, What right had
the table?
theyOnce
to say,theyWe,were
theinside,
people? Mamima
My political gave curiosity,
the pot of
Two boys were playing carrom on the steps of a yoghurt
A) Earth shows greater 35 exclusiveand theanxious
of my pot of sweetmeats
solicitude for to thethe old
public
small, painted shed whichvariation
had the in density words
following than
lady. “There
Ceres does. welfare, leadswasmeno need,”
to ask, Who sheauthorized
said. “Oh really,”them toshe
on its wall in large, black letters: NATIONAL said.
speak“This is too much,”
the language of, We, shetheinsisted,
people,with insteadthe of,
air of
Line B) Juno’s average
ASSOCIATION OFdistance
SPORTSMEN.from theASun is less than
single
20 one
We, who has just
the states? received
States thecharacteristics
are the Kohinoor diamond and the as
that of Ceres.
5 table-tennis table inside the shed could be glimpsed
asoul
birthday present. “Come,
of a confederation. If thecome,
statescome,”
be notsaid the agents
through
C) Somethe window.
objects The
in the boys interrupted
asteroid their in
belt are greater Chhotomama,
40 of this compact,with the air
it must of someone
be one who has just
great, consolidated,
gamemassto give
thanChhotomama
Ceres is. directions to the house given thegovernment,
Kohinoor diamond as a birthday present,
national of the people of all the
in a series of sporadic, enthusiastic gestures. Oh yes, and refuses
D) states. . . . to be overawed by his own generosity.
theyNo other
knew thedwarf planetAnd
old couple. has ayes,
radius
theirasson
largeandas “It’sThenothing.” wasthem
that of Ceres. 25 people It gave nothing, of course,
no power to useonly their
10 daughter-in-law had arrived last night with their
Ganguram’s
name. That they sweets and yoghurt,
exceeded but they
their power fussed and
is perfectly
first child. fussed
45 clear. Itand created
is not merethe illusionthat
curiosity thatactuates
it was something,
me: I wish
“Is it a girl or a boy?” asked Mamima, rolling something
32 down the window. to hear the unique and untasted
real, actual, and unencountered.
existing danger, which should
leadTheus tosontake
andthose
the daughter-in-law
steps, so dangerous emergedin myshyly
“A girl,”
Which data said the boy.
presented in the table would McKinnon from the anteroom. They both stooped gently to of
30 conception. Disorders have arisen in other parts
15 findMamima
most usefulrolledto up
his her window before the
argument? touch Chhotomama’s
America; but here, sir,feet, and Sandeep’s
no dangers, aunt’s andor
no insurrection
mosquitoes came in. The two boys vanished behind his mother’s feet, a traditional greeting
A) The average 50 tumult have happened; every thing has andbeenacalmmark of
them. When theydensity
reachedofthePluto is similar
house, to thatthat
they found of
obeisance towards one’s elders.
Ceres. and tranquil. But, notwithstanding this, we are
the old man was waiting on the verandah with a “Oh no no
wandering on no,” said Chhotomama,
the great ocean of human struggling
affairs. I seeto
lantern
B) in his is
Neptune hand. Moths
located muchwere shuddering
farther from the round
Sun keep the son’stohand away fromare hisrunning
feet. “There’s no
35 no landmark guide us. We we know
20 and than
roundis the lantern, though the old man was
Ceres. need for all this.”
not whither. This was
Difference half a token
of opinion has gonegesture to a
oblivious
C) to them.
The mass He had
of Ceres come
is only out because
slightly greaterhe had
than towards
55 degree ofmodesty,
inflammatory and half towards the
resentment new,
in different parts
heard the throbbing
that of Juno. of the engine in the distance. “modern” India—Nehru’s secular India, free
of the country, which has been occasioned byofthis
ritual
The night had been silent except for the questioning and religion.
D) Ceres is denser, perilous innovation. The federal Convention ought
cry of an owl and theon average,orchestral
continual than eithersoundNeptune
of
or Uranus. 40 to have amended the old system; for this purpose
25 crickets in the bushes. The throbbing of the engine
they were solely delegated; the object of their mission
had, therefore, travelled through the silence to the
April QAS 2017

Unauthorized copying or reuse of any part of this page is illegal. 2


10 CO NTI N U E
1 1
Question-and-Answer Service

extended
“I havetonot nomet
other you
consideration.
for two years,YouDada,”
must,said the 3
33

...............................................................................................................................................................................................................................................................................................................
therefore,
son, strugglingforgiveto the
get his
solicitation
hands near
of one
Chhotomama’s
unworthy
60 member
toes. “You tomust
knownot what
stop
danger
me.” This
couldwas
havehalf
arisen
a token As Passage
In used in lines 37 and
1, Henry 40,that
states “air”Virginia
most nearly means
differs from
45 under
gesturethe towards
present modesty,
Confederation,
and halfand
towards
what the
are old,
the other areas of the country in that
A) atmosphere.
causes
“traditional”
of thisIndia—Gandhi’s
proposal to change India
ourofgovernment.
ceremony and A) there is no evidence of civil unrest.
custom. B) absence.
Passage 2 B) the federal convention is widely praised.
Sandeep, meanwhile, had come to the conclusion C) demeanor.
Mr. Chairman, my worthy friend (Mr. Henry) C) opposition
65 that the grown-ups were mad, each after his or her
has expressed great uneasiness in his mind, and D) melody. to a consolidated government is
own fashion. Simple situations were turned into strong.
informed us that a great many of our citizens are also
complex, dramatic ones; not until then did everyone D) tyranny and loss of liberty are greatly feared.
50 extremely uneasy, at the proposal of changing our
feel important and happy. Will they never grow up? 4
government. . . .
thought Sandeep irately. He glanced around him. A
. . . [A]n objection is made to the form: the The characters’ behavior during the gift giving
70 single blue, fluorescent tube was burning on the wall. 34
expression, We, the people, is thought improper. mainly serves to
It was not a big room. Despite its bareness, the
Permit me to ask the gentleman who made this The figurative language in lines 33-35 (“But,
impression it gave was of austerity rather than A) emphasize the
55 objection, who but the people can delegate powers?
poverty. It made one remember that poverty meant notwithstanding . . lavish
. guidevalue of the gift.
us”) serves mainly to
Who but the people have a right to form B) inflate the significance of the gesture.
suggest
displacement as well as lack, while austerity meant
government? The expression is a common one, and a
75 being poor in a rooted way, within a tradition and C) convey indifference toward the gift.
favorite one with me. The representatives of the A) impatience.
culture of sparseness, which transformed even the D) stress the need for polite behavior.
people, by their authority, is a mode wholly B) uncertainty.
lack, the paucity, into a kind of being.
60 inessential. If the objection be, that the Union ought
C) optimism.
to be not of the people, but of the state governments,
then I think the choice of the former very happy and 5 D) indignation.
1 proper. What have the state governments to do
Which choice provides the best evidence for the
with it? . . . to the passage, the old man was standing
According answer to the previous question?
65 on But the powerbecause
the verandah of the Convention is doubted. 35
What is the power? To propose, not to determine. A) Lines 43-44
In Passage (“It wasindicates
2, Pendleton . . . yoghurt”)
that the phrase
A)
Thishe was watching
power of proposingcarswas
travel down
very theitroad.
broad; extended B) Lines
“We, 44-46 (“they
the people” is . . . unencountered”)
to
B) remove
the twoallboysdefects
hadin government:
reported the members
the visitors would of
that soon
Convention,
arrive. who were to consider all the defects C)
A) Lines 52-54 (“Oh
objectionable no . .citizens.
to most . all this”)
70 in our general government, were not confined to any
C) he had heard what he believed to be the D)
B) Lines 58-60 (“I
appropriate for have . . . stop me”)
the proposed type of
particular
visitors’plan.
car.Were they deceived? This is the government.
proper question here. Suppose the paper on your
D) he
table enjoyedfrom
dropped listening
one oftothe
theplanets;
quiet sounds of
the people 6 C) popular among proponents of states’ rights.
found theit,evening.
and sent us here to consider whether it was D) overused by supporters of a federal system.
The description of Chhotomama and the son’s
75 proper for their adoption; must we not obey them?
interaction mainly serves to
Then the question must be between this government
2 and the Confederation. The latter is no government 36 A) show how the characters diverge in their
at
In all.
theItpassage,
has been thesaid that itand
yoghurt hassweetmeats
carried us, through
are a approaches to cultural practices.
As used in line 79, “issue” most nearly means
dangerous
compared to a war, to a happy issue. Not that
B) emphasize the characters’ complex relationship.
80 Confederation, but common danger, and the spirit of A) misgiving.
A) jewel.were bonds of our union: union and
America, C) stress the characters’ misinterpretations of
B) publication.
Indian history.
unanimity,
B) cuisine.and not that insignificant paper, carried
us through that dangerous war. “United, we C) proceeding.
D) depict how the characters created gestures that
C) wedding gift.
stand—divided, we fall!” echoed and reëchoed D) became
outcome.routine.
D) generous
85 through donation. Congress to the drunken
America—from
carpenter—was effectual, and procured the end of
our wishes, though now forgotten by gentlemen, if
such there be, who incline to let go this stronghold,
to catch at feathers; for such all substituted projects
90 may prove.

April QAS 2017

Unauthorized copying or reuse of any part of this page is illegal. 11


3 CO NTI N U E
1 1
Question-and-Answer Service

37
Reading Test 40

...............................................................................................................................................................................................................................................................................................................
Based on Passage 2, which statement best reflects Which statement best expresses Henry’s and
Pendleton’s view 65
of theM I NU of
Articles TES, 5 2 QUESTIONSPendleton’s respective views of the Confederation?
Confederation?
A) They were a source of great concern to a large A) Henry felt it was flawed but correctable, while
Turn to Section
number of American citizens.1 of your answer sheet to answer the questions
Pendleton feltinthat
thisitsection.
had served no useful
B) They gave too much power to the elected purpose.
representatives. B) Henry viewed it as a perfect expression of
C) They served to unite the country in ways that democracy, while Pendleton viewed it as
were unprecedented. fundamentally authoritarian.
Each passage or pair of passages below is followed by a number of questions. After reading
D) They had little each passage
to do or pair, choose
with America’s having C) question
the best answer to each Henry regarded
based onit what
as adequate
is statedinorits current
prevailed in its most recent
implied form, while Pendleton
conflict.or passages and in any accompanying graphics (such as a table or
in the passage regarded it as a
transitional system only.
graph).
D) Henry considered it an unavoidable compromise
38 during a time of crisis, while Pendleton
considered it to have harmed the nation’s future
Which choice provides the best evidence for the
Questions 1-10 are based on the following prospects.
old man’s listening ear, and to his wife’s ear, even
.......................................................................................................................................................................................................

answer to the previous question?


passage. when the car was relatively far away and beyond their
A) Lines 47-51
This passage (“Mr.from
is adapted Chairman . . . government”)
Amit Chaudhuri, A Strange and
range of vision. They had pondered over the sound,
B) Lines
Sublime 58-60©1991
Address. (“Thebyrepresentatives
Amit Chaudhuri..A. .ten-year-old 41
30 and finally, he had lit the lantern and shuffled out.
boy named Sandeep travels with his mother, his aunt
inessential”) “I told her,” he said, referring to his wife. “I told her
Henry would most likely have responded to
(Mamima), and his uncle (Chhotomama) to visit family in that I heard the car, I knew it was the car, I told her
C) Lines 79-83 (“Not . . . war”) Pendleton’s claim about the members of the
Calcutta, India. you were coming.”
Convention by asserting that they
D) Lines 83-89 (“United . . . feathers”) Once they were inside, Mamima gave the pot of
Two boys were playing carrom on the steps of a yoghurt andsufficiently
the pot of sweetmeats
35 A) did not address theto the old
defects of the
small, painted shed which had the following words lady.Confederation.
“There was no need,” she said. “Oh really,” she
on its wall in large, black letters: NATIONAL said.should
“This isnottoohave
much,” she insisted, withnew
the air of
39 B) proposed an entirely form
Line ASSOCIATION OF SPORTSMEN. A single
one who has just
of government. received the Kohinoor diamond as
table-tennis
5 Which table best
statement inside the shedthe
describes could be glimpsed
relationship a birthday present. “Come, come, come,” said
through the window.
between The boys
views expressed in interrupted their
the two passages? 40 C) were seeking
Chhotomama, only
with thetoairenact the wishes
of someone whoofhas
the just
game to give Chhotomama directions to the house American people.
given the Kohinoor diamond as a birthday present,
A) Henryofand
in a series Pendleton
sporadic, both disagreed
enthusiastic with
gestures. Ohthe
yes,
conclusions of the federal Convention. and failed
D) refusestotounderstand
be overawed thebydanger
his own generosity.
of taking no
they knew the old couple. And yes, their son and “It’s significant was nothing, of course, only
nothing.” Itaction.
10 B) Henry and Pendleton
daughter-in-law had arrived held
lastsimilar beliefs
night with about
their Ganguram’s sweets and yoghurt, but they fussed and
the new
first child. Constitution.
45 fussed and created the illusion that it was something,
C) “Is it a girl
Henry or aquestions
asked boy?” askedthatMamima,
Pendletonrolling
admitted 42 something unique and untasted and unencountered.
down hethe window.
could not answer. The son and the daughter-in-law emerged shyly
“A girl,” said the boy. Which
from the choice provides
anteroom. They theboth
beststooped
evidencegently
for the
to
15
D) Pendleton disagreed with most
Mamima rolled up her window before the of the points answer to the previous question?
made by Henry. touch Chhotomama’s feet, and Sandeep’s aunt’s and
mosquitoes came in. The two boys vanished behind his mother’s
50 A) feet, a traditional greeting and a mark of
them. When they reached the house, they found that Lines 6-10 (“And here . . . confederation”)
obeisance towards one’s elders.
the old man was waiting on the verandah with a B) Lines 11-13 (“That . . . striking”)
“Oh no no no,” said Chhotomama, struggling to
lantern in his hand. Moths were shuddering round C)
keepLines 39-42
the son’s (“The
hand federal
away from. .his. consideration”)
feet. “There’s no
20 and round the lantern, though the old man was
needLines
D) for all42-46
this.”(“You
This was
. . . half a token gesture
government”)
oblivious to them. He had come out because he had 55 towards modesty, and half towards the new,
heard the throbbing of the engine in the distance. “modern” India—Nehru’s secular India, free of ritual
The night had been silent except for the questioning and religion.
cry of an owl and the continual orchestral sound of
25 crickets in the bushes. The throbbing of the engine
had, therefore, travelled through the silence to the
April QAS 2017

Unauthorized copying or reuse of any part of this page is illegal. 2


12 CO NTI N U E
1 1
Question-and-Answer Service

Questions 43-52
“I have not metare
youbased onyears,
for two the following
Dada,” said the 3 John Ralph and colleagues at the

...............................................................................................................................................................................................................................................................................................................
passage.
son, struggling to get his hands near Chhotomama’s University of Wisconsin-Madison’s Great Lakes
60 toes. “You must not stop me.” This was half a token As used in Research
Bioenergy lines 37 and 40, “air”
Center mostlignin
detected nearlyinmeans
This passage is adapted from Catherine Clabby, “A Tangled
gesture
Tale towards
of Plant modesty,
Evolution.” and
©2009 by half
Sigmatowards the old,
Xi, The Scientific 45 C.
A) cheilosporioides.
atmosphere. They found the same telltale
“traditional” India—Gandhi’s
Research Society. India of ceremony and components derived from radical coupling reactions
custom. B) absence.
of hydroxycinnamyl alcohols used to describe lignins
As ancestors
Sandeep, of land plants
meanwhile, abandoned
had come their
to the conclusion C) terrestrial
in demeanor. plants.
aquatic nurseries for life on shore, they
65 that the grown-ups were mad, each after his or her needed the
D) At the Centre de Recherches sur les
melody.
means to seal in water and hold themselves
own fashion. Simple situations were turned into up to 50 Macromolécules Végétales in France, Katia Ruel
Line thrive.
complex, Lignin, a strengthening
dramatic ones; not untilandthenstiffening
did everyone applied antibodies designed to locate lignin within
5 polymer common in woody plant
feel important and happy. Will they never cells, contributes
grow up? 4 land plants to samples of C. cheilosporioides. Her
to both extremely well.
thought Sandeep irately. He glanced around him. A tests detected lignin in the seaweed too.
Lignin
70 single blue,production
fluorescentfor tubethose
wastasks was considered
burning on the wall. TheThe
characters’
amountsbehavior
are muchduring smaller thethan
gift what
givingis
aItkey adaptive achievement of vascular
was not a big room. Despite its bareness, plants,
the which mainly serves to
55 found in land plants. But lignin is most abundant in
descend from green algae. Now a University
impression it gave was of austerity rather than of the
10 British Columbia botanist and some highly A) parts of the the
emphasize seaweed
lavishthat
value areofmost mechanically
the gift.
poverty. It made one remember that poverty meant stressed, which suggests to Martone that there could
specialized
displacement chemists
as well have strong
as lack, while evidence
austerityformeant
lignin B) inflate
be some the significance
environmental of the gesture.
stimulation that increases
in a red alga called Calliarthron cheilosporioides.
75 being poor in a rooted way, within a tradition and production
C) convey of the polymer
indifference in thethe
toward organism.
gift. The
The finding
culture suggests
of sparseness, that transformed
which a biological building
even the 60 puzzling thing is that it’s also present in calcified
block fundamental to the success of land plants has D) stress the need for polite behavior.
lack, the paucity, into a kind of being. portions of the algae. “We don’t know what it’s doing
15 roots that stretch back far deeper—and maybe there,” Martone says.
wider—through evolutionary time than was known. Martone’s working hypothesis is that the
5
1 “This pathway is involved in the production of other molecular pathways producing lignin emerged long
secondary metabolites like pigments in plants. A lot Which land
65 before choice provides
plants evolvedthefrombest evidence for back
green algae, the to
According
of to the
that is likely to passage,
be conservedthe old manfar
pretty wasback
standing
in the to the previous
answerancestor
some shared withquestion?
red algae more than a
on the verandah
20 evolutionary because
history of algae,” says Patrick T. billion years ago. Molecular evidence and
Martone, the botanist who led the study. A) Lines 43-44 (“It was . . . yoghurt”)
A) he was watching cars travel down the road. comparisons of the biological gear the algae use to
Martone didn’t set out to locate lignin in algae. B) Lines
harvest 44-46
light (“theyhim
convince . . . unencountered”)
that both red and green
B) the
The two boys had
biomechanist reported
simply wanted thetovisitors
better would 70 algae descend from one
soon arrive. C) Lines 52-54 (“Oh noendosymbiotic
. . . all this”) event, when a
understand the toughness of C. cheilosporioides, eukaryote cell engulfed a photosynthesizing
25 which
C) hedwells
had heardin the harsh
what he habitat
believedoftointertidal
be the zones D) Lines 58-60 (“I have . . . stop me”)
cyanobacterium and gained the ability to make its
along rocky shores.
visitors’ car. own food.
During high tides, waves pummel the alga with Karl J. Niklas, a Cornell University botanist,
D) he enjoyed listening to the quiet sounds of 6
water velocities exceeding 20 meters per second and 75 considers Martone’s evidence for lignin in
the evening.
with forces that exceed those generated by hurricane C.
Thecheilosporioides
description of exceptionally
Chhotomamastrong. and theBut he
son’s
30 winds. The calcified, or rigid-bodied, seaweed has thinks that red
interaction andserves
mainly green to algae evolved from separate
multiple noncalcified joints that make it flexible yet endosymbiotic events. Still, the progenitors of the
2 A) show how both
the characters diverge
strong enough to handle that setting. two algae may have carried genesinsimilar
their to
When
In the collaborator
passage, Jose Estevez
the yoghurt at the Carnegie
and sweetmeats are approaches to cultural practices.
80 those participating in the lignin production pathways
Institution
compared to fora Science examined the joints for B) emphasize
seen the characters’ complex relationship.
today, he says.
35 Martone with a transmission electron microscope, he
A) jewel. C) stress the characters’ misinterpretations of
saw secondary cell walls, features commonly found
B) land
cuisine. Indian history.
in plants. That prompted Martone and Estevez
to seek
C) weddingout experts
gift. in lignin, a molecule of great D) depict how the characters created gestures that
research interest right now because its toughness became routine.
D) generous donation.
40 impedes the use of some plants as sources of biofuel
and animal feed.

April QAS 2017

Unauthorized copying or reuse of any part of this page is illegal. 13


3 CO NTI N U E
1 1
Question-and-Answer Service

43
Reading Test 46

...............................................................................................................................................................................................................................................................................................................
The passage is primarily concerned with Which choice provides the best evidence for the
A)
65 M I NU TES, 5 2 QUESTIONSanswer to the previous question?
narrating how a finding was arrived at and
indicating possible implications. A) Lines 7-9 (“Lignin . . . algae”)
Turn to Section 1 of your answer sheet to answer the questions in this section.
B) explaining some differences among specialists in B) Lines 13-16 (“The finding . . . known”)
different fields of science. C) Lines 23-26 (“The biomechanist . . . shores”)
C) identifying a particularly vexing phenomenon D) Lines 59-61 (“The puzzling . . . algae”)
and endorsing a single explanation.
Each
D) describing the passageoforan
properties pair of passages
organism and below is followed by a number of questions. After reading
showing howeachtheypassage or pair, choose the best answer to47each question based on what is stated or
can be exploited.
implied in the passage or passages and in any accompanying graphics (such as a table or
The passage indicates that the structure of
graph). C. cheilosporioides consists of components that
44
As presented in the passage, the conclusion that A) change density in response to external
lignin is present in C. cheilosporioides can best be conditions.
Questions 1-10 are based on the following
B) protectlistening
old man’s the cell ear, and from
to hisexposure
wife’s ear,toeven
.......................................................................................................................................................................................................

described as interior
passage. whenseawater.
the car was relatively far away and beyond their
A) theorized
This passage previously
is adapted fromwithin the largerAscientific
Amit Chaudhuri, Strange and
range of vision. They had pondered over the sound,
community.
Sublime Address. ©1991 by Amit Chaudhuri. A ten-year-old 30 C)
and regenerate
finally, he had as the
lit seaweed
the lantern colony matures.out.
and shuffled
boy named
B) foundedSandeep travels with
on empirical his mother,
evidence his aunt
and thus “I told
D) her,” together
operate he said, referring
to allow forto his wife. “I told her
suppleness.
(Mamima), and his uncle (Chhotomama) to visit family in
persuasive. that I heard the car, I knew it was the car, I told her
Calcutta, India. you were coming.”
C) certain to disprove most earlier theories of algal 48 Once they were inside, Mamima gave the pot of
Two boys were playing carrom on the steps of a
evolution. 35 yoghurt and the pot of sweetmeats to the old
small, painted shed which had the following words As used
lady. in line
“There was32,no“handle”
need,” shemost nearly
said. “Oh means
really,” she
D) supported by an abundance of conjectural
on its wall in large, black letters: NATIONAL said. “This is too much,” she insisted, with the air of
reports. A) train for.
Line ASSOCIATION OF SPORTSMEN. A single
one who has just received the Kohinoor diamond as
5 table-tennis table inside the shed could be glimpsed B) survive in.
a birthday present. “Come, come, come,” said
through the window. The boys interrupted their 40 C) engage in. with the air of someone who has just
Chhotomama,
45
game to give Chhotomama directions to the house givenactthe Kohinoor diamond as a birthday present,
in can
It a series of sporadic,
reasonably enthusiastic
be inferred gestures.
from the Ohthat
passage yes, D) on.
and refuses to be overawed by his own generosity.
they knew research
Martone’s the old couple.
interestAnd yes, their
in lignin sonbe
should and “It’s nothing.” It was nothing, of course, only
daughter-in-law had arrived last night with their
10 considered
first child. 49 Ganguram’s sweets and yoghurt, but they fussed and
A) “Is
unusually insightful, because it has rolling 45 fussed and created the illusion that it was something,
it a girl or a boy?” asked Mamima, Scientists
somethingwho uniquespecialized in lignin
and untasted andwere useful to
unencountered.
fundamentally
down the window. changed the way lignin is
Martone and Estevez because
The son and the daughter-in-law emerged shyly
understood.
“A girl,” said the boy. from
A) the thetoughness
anteroom.ofThey both stooped gently
C. cheilosporioides made it to
15 B) Mamima
somewhat questionable,
rolled because
up her window Martone
before the based touch Chhotomama’s feet, and Sandeep’s
difficult to analyze without the use of specializedaunt’s and
his conclusions on an atypical sample.
mosquitoes came in. The two boys vanished behind 50 his mother’s
chemistry. feet, a traditional greeting and a mark of
them.
C) When they
properly reached
cautious, the house,
because Martone theychecked
found that
his obeisance towards one’s elders.
the old man was
findings about waiting on the verandah
C. cheilosporioides with atimes.
multiple C. cheilosporioides
B) “Oh no no no,” saidwas found to contain
Chhotomama, features
struggling to
lantern in his hand. Moths were shuddering round typically found in plants known to
keep the son’s hand away from his feet. “There’s no contain
D) initially secondary, because it was
20 and round the lantern, though the old man was
undertaken to He
support particular questions needlignin.
for all this.” This was half a token gesture
oblivious to them. had come out because he had 55 C) priormodesty,
towards researchand suggested that seaweed
half towards the new,species
about C. cheilosporioides.
heard the throbbing of the engine in the distance. contain
“modern” molecules of asecular
India—Nehru’s particularly
India,durable
free of ritual
The night had been silent except for the questioning and chemical
religion. compound.
cry of an owl and the continual orchestral sound of
25 crickets in the bushes. The throbbing of the engine D) some unexpected measurement results from tests
had, therefore, travelled through the silence to the for lignin required specialist interpretation.

April QAS 2017

Unauthorized copying or reuse of any part of this page is illegal. 2


14 CO NTI N U E
1 1
Question-and-Answer Service

50 “I have not met you for two years, Dada,” said the 3
52

...............................................................................................................................................................................................................................................................................................................
.....................................................................................................................................................................................................................................................
son, struggling to get his hands near Chhotomama’s
Which choice provides the best evidence for the
60 toes. “You must not stop me.” This was half a token As usedcounters
Niklas in lines 37 and 40, proposal
Martone’s “air” mostabout
nearly
themeans
answer
gesture to the previous
towards modesty,question?
and half towards the old, evolutionary history of red and green algae by
A) atmosphere.
suggesting that the
“traditional” India—Gandhi’s
A) Line 22 (“Martone . . . algae”) India of ceremony and
custom. B) absence.
B) Sandeep,
Lines 30-32 (“The calcified . . . to
setting”) A) major molecular pathways present in red and
meanwhile, had come the conclusion C) green
demeanor.
algae must have been in place long before
65 C)
that Lines 33-37 (“When
the grown-ups . . . plants”)
were mad, each after his or her D) their
melody.
evolutionary lines separated.
ownLines
D) fashion.
42-45Simple situations
(“John Ralph . were
. . turned into
complex, dramatic ones; not until then did everyone B) evidence uncovered in Martone’s research does
C. cheilosporioides”) not directly address the issue of when the last
feel important and happy. Will they never grow up? 4
thought Sandeep irately. He glanced around him. A common ancestor of red and green algae existed.
The characters’ behavior
C) evolutionary during
lines leading tothe
thegift
twogiving
kinds of
51 single blue, fluorescent tube was burning on the wall.
70
It was not a big room. Despite its bareness, the mainly serves to
algae might have diverged before they acquired
As used in line
impression 76, was
it gave “strong” most nearly
of austerity rathermeans
than the ability to photosynthesize.
A) emphasize the lavish value of the gift.
poverty. It
A) compelling.made one remember that poverty meant D) process by which C. cheilosporioides produces
displacement as well as lack, while austerity meant B) inflate the significance of the gesture.
lignin might be different from the process by
75 B)
beingdistinctive.
poor in a rooted way, within a tradition and C) which
conveyancestral
indifference toward
species the gift.
of algae produced it.
culture
C) of sparseness, which transformed even the
impervious. D) stress the need for polite behavior.
lack, the paucity, into a kind of being.
D) vigorous.

5
1
Which choice provides the best evidence for the
According to the passage, the old man was standing answer to the previous question?
on the verandah because
A) Lines 43-44 (“It was . . . yoghurt”)
A) he was watching cars travel down the road.
B) Lines 44-46 (“they . . . unencountered”)
B) the two boys had reported the visitors would
soon arrive. C) Lines 52-54 (“Oh no . . . all this”)

C) he had heard what he believed to be the D) Lines 58-60 (“I have . . . stop me”)
visitors’ car.
D) he enjoyed listening to the quiet sounds of 6
the evening.
The description of Chhotomama and the son’s
interaction mainly serves to
2 A) show how the characters diverge in their
In the passage, the yoghurt and sweetmeats are approaches to cultural practices.
compared to a B) emphasize the characters’ complex relationship.
A) jewel. C) stress the characters’ misinterpretations of
B) cuisine. Indian history.

STOP
C) wedding gift. D) depict how the characters created gestures that
became routine.
D) generous donation.
If you finish before time is called, you may check your work on this section only.
Do not turn to any other section.

April QAS 2017

Unauthorized copying or reuse of any part of this page is illegal. 15


3 CO NTI N U E
1
2 1
2
Question-and-Answer Service

Readingand
Writing TestLanguage Test
65 M I NU TES, 4
35 542 QUESTIONS

Turn to Section 2
1 of your answer sheet to answer the questions in this section.

Each passage below


or pair is
ofaccompanied
passages below
by ais number
followedofbyquestions.
a numberFor
of questions.
some questions,
After reading
you
eachconsider
will passagehow
or pair,
thechoose
passagethe
might
bestbe
answer
revised
to to
each
improve
question
thebased
expression
on what
of ideas.
is stated
For or
implied
other questions,
in the passage
you willorconsider
passageshow
andthe
in any
passage
accompanying
might be edited
graphics
to (such
correct
aserrors
a table
inor
graph). structure, usage, or punctuation. A passage or a question may be accompanied by
sentence
one or more graphics (such as a table or graph) that you will consider as you make revising
and editing decisions.

Questions 1-10 are based


Some on thewill
questions following old man’s
direct you to an underlined portion of a listening
passage. ear,
Otherand to his wife’s
questions will ear, even
.......................................................................................................................................................................................................

passage. whenabout
direct you to a location in a passage or ask you to think the carthewas relatively
passage as afar away and beyond their
whole.
This passage is adapted from Amit Chaudhuri, A Strange and
range of vision. They had pondered over the sound,
Afterbyreading
Sublime Address. ©1991 each passage,
Amit Chaudhuri. choose the answer 30
A ten-year-old to and
eachfinally, he that
question had lit
mosttheeffectively
lantern and shuffled out.
improves the quality of writing
boy named Sandeep travels with his mother, his aunt in the passage or “I
that told her,”
makes he
the said,
passagereferring
conform totohisthe
wife. “I told her
conventions
(Mamima), and his uncle of standard
(Chhotomama) to visitwritten that I heard
family inEnglish. Many questions theacar,
include “NOI CHANGE”
knew it was the car, I told her
option.
Calcutta, India. Choose that option if you think the best choice is to you were
leave coming.”
the relevant portion of the
Once they were inside, Mamima gave the pot of
Two boys werepassageplayingascarrom
it is. on the steps of a
35 yoghurt and the pot of sweetmeats to the old
small, painted shed which had the following words lady. “There was no need,” she said. “Oh really,” she
on its wall in large, black letters: NATIONAL said. “This is too much,” she insisted, with the air of
Line ASSOCIATION OF SPORTSMEN. A single
Questions 1-11 are based on the following passage. 1 one who has just received the Kohinoor diamond as
...................................................................................................................................

5 table-tennis table inside the shed could be glimpsed


a birthday present. “Come, come, come,” said
through the window. The boys interrupted their A) NO CHANGE
40 Chhotomama, with the air of someone who has just
game to give Chhotomama directions to the house given
The inCase for Electronic
a series of sporadic, Health Records
enthusiastic gestures. Oh yes, B) prescribed; and diamond as a birthday present,
the Kohinoor
and refuses to be overawed by his own generosity.
they knew the old couple.
The US health-care system has madeAnd yes, their son and
significant C) prescribed:
“It’s nothing.” Itand
was nothing, of course, only
10 daughter-in-law had arrived last night with their
D) prescribed
Ganguram’s and, and yoghurt, but they fussed and
sweets
strides
firstinchild.
the implementation of systems that manage
45 fussed and created the illusion that it was something,
“Is it a girl or a boy?” asked Mamima, rolling something unique and untasted and unencountered.
electronic
down health records, which include information
the window. The son and the daughter-in-law emerged shyly
such as “A girl,” said
a patient’s the boy.
medical history, medications currently from the anteroom. They both stooped gently to
15 Mamima rolled up her window before the touch Chhotomama’s feet, and Sandeep’s aunt’s and
1 prescribed,
mosquitoesand came a list
in. of
Theallergies.
two boys From 2001 to
vanished behind 50 his mother’s feet, a traditional greeting and a mark of
them. When they reached the house, they found that obeisance towards one’s elders.
2013,the
theolduseman
of electronic
was waiting health record
on the systems
verandah bya
with “Oh no no no,” said Chhotomama, struggling to
lantern in his hand. Moths were shuddering round keep the son’s hand away from his feet. “There’s no
20 and round the lantern, though the old man was
need for all this.” This was half a token gesture
oblivious to them. He had come out because he had 55 towards modesty, and half towards the new,
heard the throbbing of the engine in the distance. “modern” India—Nehru’s secular India, free of ritual
The night had been silent except for the questioning and religion.
cry of an owl and the continual orchestral sound of
25 crickets in the bushes. The throbbing of the engine
had, therefore, travelled through the silence to the
April QAS 2017

Unauthorized copying or reuse of any part of this page is illegal. 2


16 CO NTI N U E
1
2 1
2
Question-and-Answer Service

office-based physicians
“I have 2 for
not met you began
twotoyears,
decline only said
Dada,” in the
the 32

...............................................................................................................................................................................................................................................................................................................
son, struggling to get his hands near Chhotomama’s
last
60 three years.must
toes. “You While this
not progress
stop is commendable,
me.” This was half a tokenthe Theused
As writer wants37toand
in lines complete themost
40, “air” sentence with
nearly means
gesture towards modesty, accurate and relevant information from the graph to
US health-care system needs toand halfthe
make towards the old, to
full transition A) atmosphere.
support the claim made about advances in the
“traditional” India—Gandhi’s India of ceremony and
electronic
custom. health records a high priority. B) absence.
implementation of electronic health record systems.
Sandeep, meanwhile, had come to the conclusion Which choice
C) demeanor. best accomplishes this goal?
65 that the grown-ups were mad, each after his or her Percentage of Office-Based Physicians with
D) melody.
own fashion. Simple situations were turned into Electronic Health Record Systems, 2001–2013
complex, dramatic ones; not until then did everyone
feel important and happy. Will they never grow up? 80
4
thought Sandeep irately. He glanced around him. A
70 single blue, fluorescent tube was burning on the wall. The60
characters’ behavior during the gift giving

Percent
It was not a big room. Despite its bareness, the 40 serves to
mainly
impression it gave was of austerity rather than A) emphasize the lavish value of the gift.
poverty. It made one remember that poverty meant 20
displacement as well as lack, while austerity meant B) inflate the significance of the gesture.
75 being poor in a rooted way, within a tradition and
0
C) convey indifference toward the gift.
culture of sparseness, which transformed even the 01 002 003 004 005 006 007 008 009 010 011 012 013
20 the
D) stress 2 2 for
2 need 2 2 behavior.
2 polite 2 2 2 2 2 2
lack, the paucity, into a kind of being.
Adapted from National Center for Health Statistics, Data Brief
Number 143, US Department of Health and Human Services, Centers
5 for Disease Control and Prevention. Published in 2014.
1
Which
A) NOchoice
CHANGEprovides the best evidence for the
According to the passage, the old man was standing answer to the previous question?
on the verandah because B) increased most dramatically between 2011 and
A) 2012.
Lines 43-44 (“It was . . . yoghurt”)
A) he was watching cars travel down the road.
C) Lines
B) increased
44-46from less. than
(“they 20 percent of physicians
. . unencountered”)
B) the two boys had reported the visitors would
soon arrive. C) to nearly
Lines 80 (“Oh
52-54 percent
noof . . physicians.
. all this”)
D) Lines
D) fluctuated
58-60from year .to. . year
(“I have stopuntil
me”)about
C) he had heard what he believed to be the 80 percent of physicians were using electronic
visitors’ car. health records.
D) he enjoyed listening to the quiet sounds of 6
the evening.
The description of Chhotomama and the son’s
interaction mainly serves to
2 A) show how the characters diverge in their
In the passage, the yoghurt and sweetmeats are approaches to cultural practices.
compared to a B) emphasize the characters’ complex relationship.
A) jewel. C) stress the characters’ misinterpretations of
B) cuisine. Indian history.

C) wedding gift. D) depict how the characters created gestures that


became routine.
D) generous donation.

April QAS 2017

Unauthorized copying or reuse of any part of this page is illegal. 17


3 CO NTI N U E
1
2 1
2
Question-and-Answer Service

Reading Test
3 Regrettably, electronic medical records require 3

...............................................................................................................................................................................................................................................................................................................
infrastructure that can be expensive to build: they don’t Which phrase most effectively sets up the examples
65 M I NU TES, 5 2 QUESTIONSin the second part of the sentence?
require physical storage space, they don’t need to be
A) NO CHANGE
photocopied and collated,
Turn and 4 they
to Section areyour
1 of less likely
answer to sheet to answer the questions in this section.
B) Electronic health records provide many
be physically misplaced. More importantly, electronic advantages over paper ones:
C) Researchers have weighed the benefits and
records accelerate communication between 5 different drawbacks of electronic health records:
and various health-care providers,
Each passage allowing
or pair offor more below is followedD)
passages by aTypically,
number of questions.
electronic Afterrecords
health readingneed a
full-timebased
each passage or pair, choose the best answer to each question staff toonmaintain them: or
what is stated
efficient patient treatment. For example, when
implied in the passage or passages and in any accompanying graphics (such as a table or
paramedics have access graph).
to electronic records in
4
ambulances, they can learn what kinds of treatment they
A) NO CHANGE
should immediately begin on a 6 patient. Immediate
Questions 1-10 are based on the following B) man’s
old because they areear, and to his wife’s ear, even
listening
.......................................................................................................................................................................................................

passage.
treatment results in safer and earlier care. Even small when
C) being was relatively far away and beyond their
the car
This passage is adapted from Amit Chaudhuri, A Strange and
range of vision. They had pondered over the sound,
improvements in efficiency add up. The Veterans Health 30 D) DELETE
and finally, hethehadunderlined portion.
lit the lantern and shuffled out.
Sublime Address. ©1991 by Amit Chaudhuri. A ten-year-old
boy named Sandeep travels with his mother, his aunt “I told her,” he said, referring to his wife. “I told her
Administration, the largest integrated health-care system
(Mamima), and his uncle (Chhotomama) to visit family in that I heard the car, I knew it was the car, I told her
in the UnitedIndia.
Calcutta, States, reports that after the 5 you were coming.”
Two boysofwere
implementation playinghealth
electronic carrom on theinsteps
records 1995,ofits
a A) Once they were inside, Mamima gave the pot of
NO CHANGE
35 yoghurt and the pot of sweetmeats to the old
small, painted shed which had the following words B) different
lady. “There was no need,” she said. “Oh really,” she
totalon
productivity has increased
its wall in large, by 6 percent
black letters: NATIONALper year.
said. “This isdifferent
C) diverse, too much,” she insisted, with the air of
Line ASSOCIATION OF SPORTSMEN. A single
one who has just
D) singularly different received the Kohinoor diamond as
5 table-tennis table inside the shed could be glimpsed
a birthday present. “Come, come, come,” said
through the window. The boys interrupted their 40 Chhotomama, with the air of someone who has just
game to give Chhotomama directions to the house given the Kohinoor diamond as a birthday present,
in a series of sporadic, enthusiastic gestures. Oh yes, 6 and refuses to be overawed by his own generosity.
they knew the old couple. And yes, their son and “It’s nothing.” It waseffectively
nothing, of course, only
Which choice most combines the
10 daughter-in-law had arrived last night with their
Ganguram’s
sentences at the sweets and yoghurt,
underlined but they fussed and
portion?
first child. 45 fussed and created the illusion that it was something,
“Is it a girl or a boy?” asked Mamima, rolling A) patient,unique
something resultingand untasted and unencountered.
down the window.
B) Thepatient
son and the results
daughter-in-law emerged shyly
“A girl,” said the boy. from the anteroom. They both stooped
15 Mamima rolled up her window before the C) patient, and those treatments resultgently to
touch Chhotomama’s feet, and Sandeep’s aunt’s and
mosquitoes came in. The two boys vanished behind D) mother’s
50 his patient because it results greeting and a mark of
feet, a traditional
them. When they reached the house, they found that obeisance towards one’s elders.
the old man was waiting on the verandah with a “Oh no no no,” said Chhotomama, struggling to
lantern in his hand. Moths were shuddering round keep the son’s hand away from his feet. “There’s no
20 and round the lantern, though the old man was
need for all this.” This was half a token gesture
oblivious to them. He had come out because he had 55 towards modesty, and half towards the new,
heard the throbbing of the engine in the distance. “modern” India—Nehru’s secular India, free of ritual
The night had been silent except for the questioning and religion.
cry of an owl and the continual orchestral sound of
25 crickets in the bushes. The throbbing of the engine
had, therefore, travelled through the silence to the
April QAS 2017

Unauthorized copying or reuse of any part of this page is illegal. 2


18 CO NTI N U E
1
2 1
2
Question-and-Answer Service

[1] “I
These
haveconcerns, however,
not met you for twoare also Dada,”
years, problems saidfor
the 37

...............................................................................................................................................................................................................................................................................................................
son, struggling to get his hands near Chhotomama’s
paper records.
60 toes. “You [2]mustDespite these
not stop clear
me.” Thisbenefits,
was half somea token As
A) used
NO in lines 37 and 40, “air” most nearly means
CHANGE
gesture towards modesty, and half towards
patients and medical professionals still harbor concerns the old, B)
A) information to provide
atmosphere.
“traditional” India—Gandhi’s India of ceremony and C) information; providing
about the potential for error and the violation of patient
custom. B) absence.
Sandeep, meanwhile, had come to the conclusion D) demeanor.
C) information, provides
privacy
65 thatwhen electronic were
the grown-ups records areeach
mad, used. [3] his
after By noor her D) melody.
own fashion. Simple situations were
means free from errors, handwritten records are turned into
complex, dramatic ones; not until then did everyone 8
especially prone to and
feel important errors resulting
happy. Will from illegible
they never grow up? 4 A) NO CHANGE
thought Sandeep irately. He glanced around him. A
handwriting.
70 single blue,[4]fluorescent
Electronic tube
health record
was systems
burning on the canwall. B) Still,
The characters’ behavior during the gift giving
It was not aerrors
big room. Despite its bareness, the mainly serves
C) In this to
case,
actually reduce by, for instance, cross-referencing
impression it gave was of austerity rather than A)
D) emphasize
In fact, the lavish value of the gift.
7 information,
drugpoverty. It made one thisremember
provides doctors with meant
that poverty
displacement as well as lack, while austerity meant B) inflate the significance of the gesture.
automatic warnings about possible adverse drug
75 being poor in a rooted way, within a tradition and C) convey indifference toward the gift.
culture of[5] sparseness, which 9
interactions. 8 Likewise, at transformed
Brigham andeven Women’sthe D) stress the need for polite behavior.
lack, the paucity, into a kind of being. A) NO CHANGE
Hospital in Boston, Massachusetts, serious medical errors
B) are
decreased by 55 percent after an electronic record system 5
1 C) are,
was According
implemented. [6]passage,
Furthermore, Which
D) are;choice provides the best evidence for the
to the the oldpatient
man wasprivacy is no
standing answer to the previous question?
the verandah
moreonthreatened because records than it is by paper
by electronic
A) Lines 43-44 (“It was . . . yoghurt”)
A) he
records, was watching
which cars traveltodown
9 are—according theDepartment
the US road. 10 B) Lines 44-46 (“they . . . unencountered”)
B) the two boys had reported the visitors would To make
of Healthsoon
and arrive.
Human Services, typically accessed by at C) Linesthis paragraph
52-54 (“Oh nomost
. . . alllogical,
this”) sentence 2
should be placed
D) Lines 58-60 (“I have . . . stop me”)
leastC)
150he had heard
different what he professionals.
health-care 10
believed to be the
visitors’ car. A) where it is now.
The best way to address these concerns about B) before sentence 1.
D) he enjoyed listening to the quiet sounds of 6
accuracythe privacy 11 are not to avoid adopting
andevening. C) after sentence 5.
The description of Chhotomama and the son’s
D) after sentence 6.
electronic health record systems but rather to implement interaction mainly serves to
2 effectively. The benefits of fully transitioning from
them A) show how the characters diverge in their
In the passage, the yoghurt and sweetmeats are 11 approaches to cultural practices.
paper to electronic health records far outweigh any
compared to a B) emphasize the characters’ complex relationship.
A) NO CHANGE
perceived disadvantages.
A) jewel. C) stress the characters’ misinterpretations of
B) have been
B) cuisine. Indian history.
C) had been
C) wedding gift. D) depict how the characters created gestures that
D) became
is routine.
D) generous donation.

April QAS 2017

Unauthorized copying or reuse of any part of this page is illegal. 19


3 CO NTI N U E
1
2 1
2
Question-and-Answer Service

Reading Test
Questions 12-22 are based on the following passage 12

...............................................................................................................................................................................................................................................................................................................
and supplementary material.
A) NO CHANGE
65 M I NU TES, 5 2 QUESTIONSB) vital: to the sustainability of the others
The Beaver as Ecosystem Engineer C) vital, to the sustainability of the others,
Turn to Section 1 of your answer sheet to answer the questions in this section.
[1] An ecosystem is a complex web of interactions D) vital; to the sustainability of the others
between organisms and their habitats. [2] Each
component is 12 vital to the sustainability of the others 13
Each passage or pair of passages below is followedA)by aNO
number of questions. After reading
CHANGE
and to the system as a whole. [3] Beavers, large rodents
each passage or pair, choose the best answer to each question based on what is stated or
B) interdependent
that use sticks, mud, and leaves
implied in to
thebuild dams
passage orin streams,and in any accompanying graphics (such as a table or
passages
graph). C) societal
are perhaps one of the best examples of the
D) associative
13 interpersonal relationship among all aspects of an
ecosystem. [4] New
Questions 1-10efforts to cultivate
are based beaver populations
on the following 14 old man’s listening ear, and to his wife’s ear, even
.......................................................................................................................................................................................................

passage. when the car was relatively far away and beyond their
reflect a growing recognition of the vital role beaver dams To make
range this paragraph
of vision. They hadmost logical,over
pondered sentence 4
the sound,
This passage is adapted from Amit Chaudhuri, A Strange and
should
30 and be placed
finally, he had lit the lantern and shuffled out.
play Sublime
in combating
Address.the effects
©1991 of drought
by Amit andA preserving
Chaudhuri. ten-year-old
boy named Sandeep travels with his mother, his aunt “I
A)told her,”ithe
where said, referring to his wife. “I told her
is now.
species diversity.
(Mamima), and[5]
his Beavers fell trees to to
uncle (Chhotomama) build
visittheir
familydams,
in that I heard the car, I knew it was the car, I told her
Calcutta, India. B) after
you were sentence
coming.”1.
and the ponds that form behind a dam can flood the C) Once
after they were5.inside, Mamima gave the pot of
sentence
Two boys were playing carrom on the steps of a 35 yoghurt and the pot of sweetmeats to the old
surrounding area. [6]
small, painted shedDespite
whichthe hadseeming drawbacks
the following wordsto D) after sentence 6.
lady. “There was no need,” she said. “Oh really,” she
on dams,
beaver its wallemerging
in large, black letters:
research NATIONAL
confirms that they said. “This is too much,” she insisted, with the air of
Line ASSOCIATION OF SPORTSMEN. A single
one who has just received the Kohinoor diamond as
5 table-tennis
actually table insideenvironmental
provide significant the shed couldbenefits.
be glimpsed
14 15 a birthday present. “Come, come, come,” said
through the window. The boys interrupted their 40 Chhotomama, with the air of someone who has just
Scientists
game to give have called the beaver
Chhotomama an “ecosystem
directions to the house A) NO CHANGE
given the Kohinoor diamond as a birthday present,
in a series of sporadic, enthusiastic gestures. Oh yes, B) streams; dams
engineer” because its dams can alter the ecological and refuses to be overawed by his own generosity.
they knew the old couple. And yes, their son and C) streams,
“It’s nothing.”damsIt was nothing, of course, only
10 daughter-in-law
makeup of its habitat.had
By arrived lastwater
regulating nightflow
within
their Ganguram’s
D) streams—dams sweets and yoghurt, but they fussed and
first child. 45 fussed and created the illusion that it was something,
15 streams.
“Is it aDams
girl orhave ripple
a boy?” effects
asked that radiate
Mamima, to
rolling something unique and untasted and unencountered.
down the window. The son and the daughter-in-law emerged shyly
“A girl,” said the boy. from the anteroom. They both stooped gently to
15 Mamima rolled up her window before the touch Chhotomama’s feet, and Sandeep’s aunt’s and
mosquitoes came in. The two boys vanished behind 50 his mother’s feet, a traditional greeting and a mark of
them. When they reached the house, they found that obeisance towards one’s elders.
the old man was waiting on the verandah with a “Oh no no no,” said Chhotomama, struggling to
lantern in his hand. Moths were shuddering round keep the son’s hand away from his feet. “There’s no
20 and round the lantern, though the old man was
need for all this.” This was half a token gesture
oblivious to them. He had come out because he had 55 towards modesty, and half towards the new,
heard the throbbing of the engine in the distance. “modern” India—Nehru’s secular India, free of ritual
The night had been silent except for the questioning and religion.
cry of an owl and the continual orchestral sound of
25 crickets in the bushes. The throbbing of the engine
had, therefore, travelled through the silence to the
April QAS 2017

Unauthorized copying or reuse of any part of this page is illegal. 2


20 CO NTI N U E
1
2 1
2
Question-and-Answer Service

“I have not
all surrounding met you for
organisms. 16 two
Theyears,
pondsDada,” said the
that form 16
3

...............................................................................................................................................................................................................................................................................................................
son, struggling to get his hands near Chhotomama’s
as60a result of dams
toes. “You mustretain nutrient-rich
not stop me.” Thissediment,
was half a token At this
As usedpoint, the37writer
in lines is considering
and 40, addingmeans
“air” most nearly the
gesture towards modesty, and half towards the old, following sentence.
17 fostering a variety of plant life and multiplying A) atmosphere.
“traditional” India—Gandhi’s India of ceremony and
B) A beaver’s life span averages 10–12 years.
absence.
foodcustom.
sources such as plankton. Attracted by these
Sandeep, meanwhile, had come to the conclusion Should
C) the writer make this addition here?
demeanor.
resources,
65 that the many different
grown-ups kinds
were of wildlife
mad, 18hisinhabits
each after or her D)
A) melody.
Yes, because it reinforces the scientists’
own fashion. Simple situations were turned into
beaver ponds. characterization of beavers as “ecosystem
complex, dramatic ones; not until then did everyone
engineers.”
feel important and happy. Will they never grow up? 4
thought Sandeep irately. He glanced around him. A B) Yes, because it clarifies information in the
70 single blue, fluorescent tube was burning on the wall. The previous
characters’ behavior
sentence during
about the the gift giving
impact of beavers.
It was not a big room. Despite its bareness, the mainly serves to
C) No, because the detail is not necessary for
impression it gave was of austerity rather than A) understanding beavers’
emphasize the lavish impact
value ongift.
of the their
poverty. It made one remember that poverty meant ecosystems.
displacement as well as lack, while austerity meant B) inflate the significance of the gesture.
75 being poor in a rooted way, within a tradition and D) convey
C) No, because it does not
indifference provide
toward enough
the gift.
culture of sparseness, which transformed even the explanation of the factors affecting the beaver’s
D) stress the need for polite behavior.
lack, the paucity, into a kind of being. life span.

5
1 17
Which choice provides the best evidence for the
According to the passage, the old man was standing A) NOtoCHANGE
answer the previous question?
on the verandah because B) adopting
A) Lines 43-44 (“It was . . . yoghurt”)
A) he was watching cars travel down the road. C) raising
B) Lines 44-46 (“they . . . unencountered”)
B) the two boys had reported the visitors would D) rearing
soon arrive. C) Lines 52-54 (“Oh no . . . all this”)

C) he had heard what he believed to be the D) Lines 58-60 (“I have . . . stop me”)
visitors’ car. 18
D) he enjoyed listening to the quiet sounds of 6 A) NO CHANGE
the evening.
B) inhabit
The description of Chhotomama and the son’s
C) are inhabited
interaction mainly serves to
2 D) show
A) have inhabited
how the characters diverge in their
In the passage, the yoghurt and sweetmeats are approaches to cultural practices.
compared to a B) emphasize the characters’ complex relationship.
A) jewel. C) stress the characters’ misinterpretations of
B) cuisine. Indian history.

C) wedding gift. D) depict how the characters created gestures that


became routine.
D) generous donation.

April QAS 2017

Unauthorized copying or reuse of any part of this page is illegal. 21


3 CO NTI N U E
1
2 1
2
Question-and-Answer Service

Reading Test
Recent studies suggest that beaver dams not only 19

...............................................................................................................................................................................................................................................................................................................
create diversely populated ecosystems but also A) NO CHANGE
65 M I NU TES, 5 2 QUESTIONSB) to preserve them
19 preserve them during times of environmental stress.
C) preserving them
A landmark 2008 study by to
Turn ecologists
Sectionat
1 the University
of your answerofsheet to answer the questions in this section.
D) they are preserved
Alberta found that the presence of beaver populations
can mitigate the effects of drought on wetlands. The
20
researchers observed that ponds in Elk Island National
Each passage or pair of passages below is followedWhich
by a number
choice of questions.
best After
reflects the reading provided
information
each passage or pair, choose the best answer to each
in question
figures 1 based
and 2? on what is stated or
Park in Canada that had developed active beaver colonies
implied in the passage or passages and in any accompanying graphics (such as a table or
held significantly more water during years when beavers
graph). A) NO CHANGE
B) marked decline
were present than they did during years when beavers
C) greater increase
were absent. Furthermore, a group of ponds that had not D) man’s
gradual reduction
Questions 1-10 are based on the following old listening ear, and to his wife’s ear, even
.......................................................................................................................................................................................................

beenpassage.
recolonized by beavers showed a 20 smaller when the car was relatively far away and beyond their
This passage
range of vision. They had pondered over the sound,
increase in area is
ofadapted from over
open water Amit Chaudhuri, A Strange and
the same period. 30 and finally, he had lit the lantern and shuffled out.
Sublime Address. ©1991 by Amit Chaudhuri. A ten-year-old
boy named Sandeep travels with his mother, his aunt “I told her,” he said, referring to his wife. “I told her
(Mamima), and his uncle (Chhotomama) to visit family in that I heard the car, I knew it was the car, I told her
Calcutta, India. you were coming.”
Once they were inside, Mamima gave the pot of
Two boys were playing carrom on the steps of a 35 yoghurt and the pot of sweetmeats to the old
small, painted shed which had the following words lady. “There was no need,” she said. “Oh really,” she
on its wall in large, black letters: NATIONAL said. “This is too much,” she insisted, with the air of
Line ASSOCIATION OF SPORTSMEN. A single
one who has just received the Kohinoor diamond as
5 table-tennis table inside the shed could be glimpsed
a birthday present. “Come, come, come,” said
through the window. The boys interrupted their 40 Chhotomama, with the air of someone who has just
game to give Chhotomama directions to the house given the Kohinoor diamond as a birthday present,
in a series of sporadic, enthusiastic gestures. Oh yes, and refuses to be overawed by his own generosity.
they knew the old couple. And yes, their son and “It’s nothing.” It was nothing, of course, only
10 daughter-in-law had arrived last night with their
Ganguram’s sweets and yoghurt, but they fussed and
first child. 45 fussed and created the illusion that it was something,
“Is it a girl or a boy?” asked Mamima, rolling something unique and untasted and unencountered.
down the window. The son and the daughter-in-law emerged shyly
“A girl,” said the boy. from the anteroom. They both stooped gently to
15 Mamima rolled up her window before the touch Chhotomama’s feet, and Sandeep’s aunt’s and
mosquitoes came in. The two boys vanished behind 50 his mother’s feet, a traditional greeting and a mark of
them. When they reached the house, they found that obeisance towards one’s elders.
the old man was waiting on the verandah with a “Oh no no no,” said Chhotomama, struggling to
lantern in his hand. Moths were shuddering round keep the son’s hand away from his feet. “There’s no
20 and round the lantern, though the old man was
need for all this.” This was half a token gesture
oblivious to them. He had come out because he had 55 towards modesty, and half towards the new,
heard the throbbing of the engine in the distance. “modern” India—Nehru’s secular India, free of ritual
The night had been silent except for the questioning and religion.
cry of an owl and the continual orchestral sound of
25 crickets in the bushes. The throbbing of the engine
had, therefore, travelled through the silence to the
April QAS 2017

Unauthorized copying or reuse of any part of this page is illegal. 2


22 CO NTI N U E
1
2 1
2
Question-and-Answer Service

“I have not met you for two years,


Figure 1 Dada,” said the 3

...............................................................................................................................................................................................................................................................................................................
son, struggling to get his hands near Chhotomama’s
toes. “You must As used in lines 37 and 40, “air” most nearly means
60 Elknot stopNational
Island me.” This wasPonds
Park half a token
gesture towards modesty,
Recolonized by Beavers the old,
and half towards A) atmosphere.
“traditional”
200 India—Gandhi’s India of ceremony and
custom. B) absence.
Total area of open water (hectares)

180 beavers
Sandeep, meanwhile, had come to the conclusion C) demeanor.
160 present
65 that the grown-ups were mad, each after his or her D) melody.
140 Simple situations were turned into
own fashion.
complex,120dramatic ones; not until then did everyone
feel important and happy. Will they never grow up? 4
100
thought Sandeep irately. He glanced around him. A
70 80fluorescent tube was burning on the wall.
single blue, The characters’ behavior during the gift giving
It was not60a big room. Despite its bareness, the mainly serves to
impression beavers
40it gave was of austerity rather than A) emphasize the lavish value of the gift.
poverty. It made one absent
remember that poverty meant
20 B) inflate the significance of the gesture.
displacement as well as lack, while austerity meant
75 being poor0in a rooted way, within a tradition and C) convey indifference toward the gift.
1948 1950 1996 2001
culture of sparseness, which transformed even the D) stress the need for polite behavior.
lack, the paucity, into a kindYear
of being.
ponds in Group 1
5
1
Which choice provides the best evidence for the
According to the passage,Figure
the old2 man was standing answer to the previous question?
on the verandah because
Elk Island National Park Ponds A) Lines 43-44 (“It was . . . yoghurt”)
A) he was watching cars travel down
Not Recolonized the road.
by Beavers B) Lines 44-46 (“they . . . unencountered”)
B) the two200 boys had reported the visitors would
soon arrive. C) Lines 52-54 (“Oh no . . . all this”)
Total area of open water (hectares)

180
C) he had D) Lines 58-60 (“I have . . . stop me”)
160heard what he believed to be the
visitors’ car.
140
D) he enjoyed listening to the quiet sounds of 6
120
the evening.
100 beavers The description of Chhotomama and the son’s
80 absent interaction mainly serves to
2 60 A) show how the characters diverge in their
40 thebeavers
In the passage, yoghurt and sweetmeats are approaches to cultural practices.
compared20 to a absent B) emphasize the characters’ complex relationship.
A) jewel. 0 C) stress the characters’ misinterpretations of
1950 1996 2001 1948 Indian history.
B) cuisine.
Year D) depict how the characters created gestures that
C) wedding gift.
ponds in Group 2 became routine.
D) generous donation.
Figures adapted from Glynnis A. Hood and Suzanne E. Bayley, “Beaver
(Castor canadensis) Mitigate the Effects of Climate on the Area of Open
Water in Boreal Wetlands in Western Canada.” ©2008 by Biological
Conservation.

April QAS 2017

Unauthorized copying or reuse of any part of this page is illegal. 23


3 CO NTI N U E
1
2 1
2
Question-and-Answer Service

Reading Test
21 In addition to studying wildlife, researchers are 21

...............................................................................................................................................................................................................................................................................................................
collaborating with local officials to promote beaver Which choice provides the best transition from the
65 M I NU TES, 5 2 QUESTIONSprevious paragraph?
populations in habitats where they might be beneficial.
A) NO CHANGE
Washington State’s Lands
Turn Council, a nonprofit
to Section 1 of your answer sheet to answer the questions in this section.
B) Despite the beaver’s reputation as a nuisance,
organization, has begun working with the state’s C) Spurred by these findings,
Department of Ecology to reintroduce beavers to D) Motivated by this opportunity,

10,000 miles of suitable habitat.


Each passageOfficials
or pairpredict that below is followed by a number of questions. After reading
of passages
each passage or pair, choose the best answer to22each question based on what is stated or
beaver dams could help retain more than
implied in the passage or passages and in any accompanying graphics (such as a table or
A) NO CHANGE
650 trillion gallons of graph).
springtime melted snow, which
B) best—“engineering,”
could help stabilize water levels in streams during dry
C) best: “engineering”
months. This project provides a low-cost alternative to D) man’s
best, “engineering,”
Questions 1-10 are based on the following old listening ear, and to his wife’s ear, even
.......................................................................................................................................................................................................

passage.
the construction of artificial dams, which could cost when the car was relatively far away and beyond their
This passage is adapted from Amit Chaudhuri, A Strange and
range of vision. They had pondered over the sound,
billions of dollars. Through such initiatives, beaver 30 and finally, he had lit the lantern and shuffled out.
Sublime Address. ©1991 by Amit Chaudhuri. A ten-year-old
boy named “I told her,” he said, referring to his wife. “I told her
populations areSandeep travels
doing what withdo
they his 22
mother,
best;his aunt
(Mamima), and his uncle (Chhotomama) to visit family in that I heard the car, I knew it was the car, I told her
“engineering” healthier, more stable ecosystems.
Calcutta, India. you were coming.”
Once they were inside, Mamima gave the pot of
Two boys were playing carrom on the steps of a 35 yoghurt and the pot of sweetmeats to the old
small, painted shed which had the following words lady. “There was no need,” she said. “Oh really,” she
on its wall in large, black letters: NATIONAL said. “This is too much,” she insisted, with the air of
Line ASSOCIATION OF SPORTSMEN. A single
one who has just received the Kohinoor diamond as
5 table-tennis table inside the shed could be glimpsed
a birthday present. “Come, come, come,” said
through the window. The boys interrupted their 40 Chhotomama, with the air of someone who has just
game to give Chhotomama directions to the house given the Kohinoor diamond as a birthday present,
in a series of sporadic, enthusiastic gestures. Oh yes, and refuses to be overawed by his own generosity.
they knew the old couple. And yes, their son and “It’s nothing.” It was nothing, of course, only
10 daughter-in-law had arrived last night with their
Ganguram’s sweets and yoghurt, but they fussed and
first child. 45 fussed and created the illusion that it was something,
“Is it a girl or a boy?” asked Mamima, rolling something unique and untasted and unencountered.
down the window. The son and the daughter-in-law emerged shyly
“A girl,” said the boy. from the anteroom. They both stooped gently to
15 Mamima rolled up her window before the touch Chhotomama’s feet, and Sandeep’s aunt’s and
mosquitoes came in. The two boys vanished behind 50 his mother’s feet, a traditional greeting and a mark of
them. When they reached the house, they found that obeisance towards one’s elders.
the old man was waiting on the verandah with a “Oh no no no,” said Chhotomama, struggling to
lantern in his hand. Moths were shuddering round keep the son’s hand away from his feet. “There’s no
20 and round the lantern, though the old man was
need for all this.” This was half a token gesture
oblivious to them. He had come out because he had 55 towards modesty, and half towards the new,
heard the throbbing of the engine in the distance. “modern” India—Nehru’s secular India, free of ritual
The night had been silent except for the questioning and religion.
cry of an owl and the continual orchestral sound of
25 crickets in the bushes. The throbbing of the engine
had, therefore, travelled through the silence to the
April QAS 2017

Unauthorized copying or reuse of any part of this page is illegal. 2


24 CO NTI N U E
1
2 1
2
Question-and-Answer Service

Questions 23-33
“I have notare
metbased ontwo
you for the years,
following passage.
Dada,” said the 23
3

...............................................................................................................................................................................................................................................................................................................
son, struggling to get his hands near Chhotomama’s
60 toes. “You must not stop me.” This was half a token As
A) used
NO in lines 37 and 40, “air” most nearly means
CHANGE
The gesture
Giant: Michelangelo’s
towards modesty, Victory
and half towards the old, B)
A) it’s
atmosphere.
“traditional” India—Gandhi’s India of ceremony and C) its’
Toward the end of the 1400s, as the Renaissance was
custom. B) absence.
Sandeep, meanwhile, had come tomembers
the conclusion D) demeanor.
C) their
reaching 23 its height in Florence, Italy, of the
65 that the grown-ups were mad, each after his or her D) melody.
city’sown
powerful
fashion.Wool Guild
Simple were celebrating
situations were turned their
into
complex, dramatic ones; not until then did everyone 24
recently completedand
feel important city happy.
cathedral.
WillItthey
was never
a triumph
growthat
up? 4 A) NO CHANGE
added to Florence’s reputation 24 from sophisticationA
thought Sandeep irately. He glanced around him.
70 single blue, fluorescent tube was burning on the wall. B) for
The characters’ behavior during the gift giving
and Itbeauty,
was notyetathe
bigguild
room.members were
Despite its eager tothe
bareness, C) to serves to
mainly
impression it gave was of austerity rather than A)
25 fancy it up even more. They wanted D) emphasize
with the lavish value of the gift.
poverty. It made one remember thatapoverty
series ofmeant
statues
displacement as wellexterior,
as lack, while austerity meant B) inflate the significance of the gesture.
to adorn the cathedral’s 26 placing high on
75 being poor in a rooted way, within a tradition and C) convey indifference toward the gift.
buttresses
culturesoofthat the art could
sparseness, whichbetransformed
admired from afar.
even theThe 25
D) stress the need for polite behavior.
lack, the paucity, into a kind of being. A) NO CHANGE
first result of the members’ plan brought great acclaim,
B) make it look super rich.
though not quite in the way they had anticipated. 5
1 C) increase its splendor.
Which
D) givechoice provides
it a wow factor.the best evidence for the
According to the passage, the old man was standing answer to the previous question?
on the verandah because
A) Lines 43-44 (“It was . . . yoghurt”)
A) he was watching cars travel down the road. 26 B) Lines 44-46 (“they . . . unencountered”)
B) the two boys had reported the visitors would A) Lines
NO CHANGE
soon arrive. C) 52-54 (“Oh no . . . all this”)
B) Lines
D) they were placed
58-60 (“I have . . . stop me”)
C) he had heard what he believed to be the
visitors’ car. C) which were placed
D) he enjoyed listening to the quiet sounds of D) placed
6
the evening.
The description of Chhotomama and the son’s
interaction mainly serves to
2 A) show how the characters diverge in their
In the passage, the yoghurt and sweetmeats are approaches to cultural practices.
compared to a B) emphasize the characters’ complex relationship.
A) jewel. C) stress the characters’ misinterpretations of
B) cuisine. Indian history.

C) wedding gift. D) depict how the characters created gestures that


became routine.
D) generous donation.

April QAS 2017

Unauthorized copying or reuse of any part of this page is illegal. 25


3 CO NTI N U E
1
2 1
2
Question-and-Answer Service

Reading Test
In 1501, guild members began the project by 27

...............................................................................................................................................................................................................................................................................................................
commissioning a statue of David, a biblical hero who had A) NO CHANGE
65 M I NU TES, 5 2 QUESTIONSB) other sculptors had previously used it,
defeated a giant named Goliath. The sculptor chosen was
C) it was used by them before,
Michelangelo, a twenty-six-year-old
Turn to Sectionartist
1 of who
youralready
answer sheet to answer the questions in this section.
D) they used it previously to begin other sculptures,
had a reputation for great talent. He was directed to use
an enormous block of marble from the cathedral’s
28
workshop to create the statue. Nicknamed “the Giant,”
Each passage or pair of passages below is followedThe
by awriter
number of questions.
is considering After reading
deleting the underlined
each passage or pair, choose the best answer to each question
sentence. based
Should onsentence
the what is stated
be oror deleted?
kept
the block had many problems. It had been quarried more
implied in the passage or passages and in any accompanying graphics (such as a table or
than forty years earliergraph).
and had started to weather from A) Kept, because it provides evidence to support the
claim that Michelangelo would succeed where
exposure to the elements. Even worse, 27 they had others had failed.
previously used it, chipping away material to rough out a B) Kept, because it includes details that explain
Questions 1-10 are based on the following why the
old man’s citizensear,
listening of Florence
and to hismight
wife’shave
ear, been
even
.......................................................................................................................................................................................................

basicpassage.
shape but giving up midtask. 28 The marble came whencritical
the carofwas
commissioning
relatively far aaway
sculptor from a their
and beyond
rangedifferent city.
of vision. They had pondered over the sound,
fromThis
thepassage is adapted
Fantiscritti fromin
quarries Amit Chaudhuri,
Carrara, A Strange
a small town and 30 and finally, he had lit it
the lantern and shuffled out.
Sublime Address. ©1991 by Amit Chaudhuri. A ten-year-old C) Deleted, because adds irrelevant information
almost
boy80 milesSandeep
named north oftravels
Florence. Michelangelo
with his was
mother, his aunt “I told
that distracts from the paragraph’s focus on her
her,” he said, referring to his wife. “I told the
(Mamima), and his uncle (Chhotomama) to visit family in that obstacles
I heard the car, I knew itfaced.
Michelangelo was the car, I told her
facedCalcutta,
with trying
India.to sculpt a monumental statue out of a you were coming.”
D) Once
Deleted,
theybecause it failsMamima
were inside, to explain why
gave thethe
potblock
of
Two
stone that wasboys were playing
generally carrom
considered on the steps of a
ruined. 35 had not already been made into
yoghurt and the pot of sweetmeats to the old a sculpture.
small, painted shed which had the following words lady. “There was no need,” she said. “Oh really,” she
on its wall in large, black letters: NATIONAL said. “This is too much,” she insisted, with the air of
Line ASSOCIATION OF SPORTSMEN. A single
one who has just received the Kohinoor diamond as
5 table-tennis table inside the shed could be glimpsed
a birthday present. “Come, come, come,” said
through the window. The boys interrupted their 40 Chhotomama, with the air of someone who has just
game to give Chhotomama directions to the house given the Kohinoor diamond as a birthday present,
in a series of sporadic, enthusiastic gestures. Oh yes, and refuses to be overawed by his own generosity.
they knew the old couple. And yes, their son and “It’s nothing.” It was nothing, of course, only
10 daughter-in-law had arrived last night with their
Ganguram’s sweets and yoghurt, but they fussed and
first child. 45 fussed and created the illusion that it was something,
“Is it a girl or a boy?” asked Mamima, rolling something unique and untasted and unencountered.
down the window. The son and the daughter-in-law emerged shyly
“A girl,” said the boy. from the anteroom. They both stooped gently to
15 Mamima rolled up her window before the touch Chhotomama’s feet, and Sandeep’s aunt’s and
mosquitoes came in. The two boys vanished behind 50 his mother’s feet, a traditional greeting and a mark of
them. When they reached the house, they found that obeisance towards one’s elders.
the old man was waiting on the verandah with a “Oh no no no,” said Chhotomama, struggling to
lantern in his hand. Moths were shuddering round keep the son’s hand away from his feet. “There’s no
20 and round the lantern, though the old man was
need for all this.” This was half a token gesture
oblivious to them. He had come out because he had 55 towards modesty, and half towards the new,
heard the throbbing of the engine in the distance. “modern” India—Nehru’s secular India, free of ritual
The night had been silent except for the questioning and religion.
cry of an owl and the continual orchestral sound of
25 crickets in the bushes. The throbbing of the engine
had, therefore, travelled through the silence to the
April QAS 2017

Unauthorized copying or reuse of any part of this page is illegal. 2


26 CO NTI N U E
1
2 1
2
Question-and-Answer Service

[1] “I
His seventeen-foot-high,
have not met you for two intricately detailed
years, Dada,” said the 29
3

...............................................................................................................................................................................................................................................................................................................
son, struggling to get his hands near Chhotomama’s
figure depicts
60 toes. “YouDavid
must notin the actme.”
stop of confronting
This was halfGoliath.
a token As
A) used
NO in lines 37 and 40, “air” most nearly means
CHANGE
gesture towards modesty, and half towards
[2] Michelangelo had solved the problem of the awkward the old, B)
A) reflected
atmosphere.
“traditional” India—Gandhi’s India of ceremony and C) had reflected
size custom.
and shape of the marble block by positioning David B) absence.
Sandeep, meanwhile, had come to the conclusion D) demeanor.
C) will reflect
turned
65 thatslightly sidewayswere
the grown-ups with mad,
his weight shifted
each after hisonto one
or her D) melody.
own fashion.
leg, poised Simple
as if ready situations
to burst were turned
into action. [3] Butinto
complex, dramatic ones; not until then did everyone 30
Michelangelo took and
feel important on the GiantWill
happy. withthey
zealnever
and finished
grow up? 4 To make this paragraph most logical, sentence 3
thought Sandeep irately. He glanced around him. A should be placedbehavior during the gift giving
the
70 statue
single in justfluorescent
blue, two years. [4]
tubeThe
wasstatue’s
burning form andwall.
on the The characters’
It was not athebig proportions
room. Despite its bareness, the mainly serves
A) where it istonow.
posture echoed of classical Roman
impression it gave was of austerity rather than B) emphasize
A) before sentence 1. value of the gift.
the lavish
sculpture,
poverty.butItits expressiveness
made one rememberand that
levelpoverty
of detailmeant
displacement as well as lack, while austerity meant C) inflate
B) after sentence 1.
the significance of the gesture.
29
75 has
beingreflected
poor inRenaissance
a rooted way,sensibilities.
within a tradition and D) convey
C) after sentence 4.
indifference toward the gift.
culture of sparseness,
[5] Michelangelo had overcome the which transformed
limitationseven the
of the D) stress the need for polite behavior.
lack, the paucity, into a kind of being.
marble block and, moreover, had turned it into a 31
technical and artistic masterpiece. 30 5 Which choice gives a second reason and additional
1
supportchoice
Which for theprovides
main idea
theofbest
the evidence
paragraph?
for the
Upon viewing
According thepassage,
to the stunning statue,
the guild
old man members
was standing answer to the
A) NO CHANGE previous question?
on thethe
discarded verandah
plan to because
hoist the statue to an exterior
A)
B) Lines 43-44 (“It
and depicted was . . .biblical
a favorite yoghurt”)story of the
A) he
buttress. was far
It was watching cars travel
too beautiful ( 31 down the unlike
and was road. citizens of Florence
B) Lines 44-46 (“they . . . unencountered”)
B) the two boys had reported the visitors would C) and later would
other Renaissance depictions of David) to be placed high
soon arrive. C) Lines 52-54 (“Ohcome
no . .to. all
symbolize
this”) Florence’s
defense of its civil liberties
C)itshe had heard D) Lines 58-60 (“I have . . . stop me”)
above viewers. Afterwhat he believed
meeting to officials
with city be the and D) and, at more than eight tons, far too heavy
visitors’ car.
D) he enjoyed listening to the quiet sounds of 6
the evening.
The description of Chhotomama and the son’s
interaction mainly serves to
2 A) show how the characters diverge in their
In the passage, the yoghurt and sweetmeats are approaches to cultural practices.
compared to a B) emphasize the characters’ complex relationship.
A) jewel. C) stress the characters’ misinterpretations of
B) cuisine. Indian history.

C) wedding gift. D) depict how the characters created gestures that


became routine.
D) generous donation.

April QAS 2017

Unauthorized copying or reuse of any part of this page is illegal. 27


3 CO NTI N U E
1
2 1
2
Question-and-Answer Service

Reading Test
prominent citizens, the members agreed that the statue 32

...............................................................................................................................................................................................................................................................................................................
should instead stand outside of Florence’s town hall A) NO CHANGE
65 M I NU TES, 5 2 QUESTIONSB) to symbolize and represent
32 as a symbol and representation of the city’s strength
C) as a symbol of
and independence. Thus,
Turnthe guild members
to Section achieved
1 of your answer sheet to answer the questions in this section.
D) as a representation that symbolized
their goal of enhancing Florence’s 33 prestige.

33
Each passage or pair of passages below is followedThe
by awriter
number of questions.
wants After
to revise the reading portion so
underlined
each passage or pair, choose the best answer to each
thatquestion based on
the concluding what issummarizes
sentence stated or the main
implied in the passage or passages and in any accompanying
ideas of the graphics
passage. (such
Whichaschoice
a tablebest
or accomplishes
graph). this goal?
A) prestige, and Michelangelo was hailed as a hero
in his own right for conquering the Giant and
Questions 1-10 are based on the following givinglistening
old man’s Florenceear,
a fitting
and tomonument.
his wife’s ear, even
.......................................................................................................................................................................................................

passage. B) prestige,
when the car and
was Michelangelo would
relatively far away andsoon leavetheir
beyond
This passage is adapted from Amit Chaudhuri, A Strange and
rangeFlorence forThey
of vision. Rome, hadwhere he would
pondered overpaint the
the sound,
Sublime Address. ©1991 by Amit Chaudhuri. A ten-year-old 30 and ceiling
finally, of
hethe
hadSistine
lit the Chapel.
lantern and shuffled out.
boy named Sandeep travels with his mother, his aunt “I
C)told her,” he
prestige, said, referring
especially to his
after parts ofwife. “I toldwere
the statue her
(Mamima), and his uncle (Chhotomama) to visit family in that decorated
I heard thewith
car, gold.
I knew it was the car, I told her
Calcutta, India. you were coming.”
D) Once
prestige,
theybut
wereeven though
inside, it boasts
Mamima works
gave of art
the pot of
Two boys were playing carrom on the steps of a 35 like Michelangelo’s David, Italy
yoghurt and the pot of sweetmeats to the oldtoday ranks only
small, painted shed which had the following words lady.fifth in terms
“There was noof need,”
revenue generated
she said. “Ohbyreally,”
tourism.
she
on its wall in large, black letters: NATIONAL said. “This is too much,” she insisted, with the air of
Line ASSOCIATION OF SPORTSMEN. A single one who has just received the Kohinoor diamond as
5 table-tennis table inside the shed could be glimpsed a birthday present. “Come, come, come,” said
through the window. The boys interrupted their 40 Chhotomama, with the air of someone who has just
game to give Chhotomama directions to the house given the Kohinoor diamond as a birthday present,
in a series of sporadic, enthusiastic gestures. Oh yes, and refuses to be overawed by his own generosity.
they knew the old couple. And yes, their son and “It’s nothing.” It was nothing, of course, only
10 daughter-in-law had arrived last night with their Ganguram’s sweets and yoghurt, but they fussed and
first child. 45 fussed and created the illusion that it was something,
“Is it a girl or a boy?” asked Mamima, rolling something unique and untasted and unencountered.
down the window. The son and the daughter-in-law emerged shyly
“A girl,” said the boy. from the anteroom. They both stooped gently to
15 Mamima rolled up her window before the touch Chhotomama’s feet, and Sandeep’s aunt’s and
mosquitoes came in. The two boys vanished behind 50 his mother’s feet, a traditional greeting and a mark of
them. When they reached the house, they found that obeisance towards one’s elders.
the old man was waiting on the verandah with a “Oh no no no,” said Chhotomama, struggling to
lantern in his hand. Moths were shuddering round keep the son’s hand away from his feet. “There’s no
20 and round the lantern, though the old man was need for all this.” This was half a token gesture
oblivious to them. He had come out because he had 55 towards modesty, and half towards the new,
heard the throbbing of the engine in the distance. “modern” India—Nehru’s secular India, free of ritual
The night had been silent except for the questioning and religion.
cry of an owl and the continual orchestral sound of
25 crickets in the bushes. The throbbing of the engine
had, therefore, travelled through the silence to the
April QAS 2017

Unauthorized copying or reuse of any part of this page is illegal. 2


28 CO NTI N U E
1
2 1
2
Question-and-Answer Service

Questions 34-44
“I have notare
metbased ontwo
you for the years,
following passage.
Dada,” said the 34
3

...............................................................................................................................................................................................................................................................................................................
son, struggling to get his hands near Chhotomama’s
60 toes. “You must not stop me.” This was half a token
Which
As usedchoice
in linesprovides the “air”
37 and 40, best introduction to the
most nearly means
Sharing
gesturefor towards
Success modesty, and half towards the old, paragraph?
A) atmosphere.
“traditional” India—Gandhi’s India of ceremony and A) absence.
NO CHANGE
34 The twenty-first century has presented both
custom. B)
B) demeanor.
C) Companies are always searching for new and
unique Sandeep,
challengesmeanwhile, had come
and innovative to the
solutions toconclusion innovative ways to recruit the best employees.
65 that the grown-ups were mad, each after his or her D) melody.
work-related
own fashion. issues. Bothsituations
Simple Julie Levine andturned
were Julie Rocco
into C) In 2007, two highly successful full-time
complex, dramatic ones; not until then did everyone engineers at a US car company faced the same
desired
feel aimportant
healthier work/life
and happy. balance, butnever
Will they they were
grow up? dilemma.
4
thought Sandeep irately. He glanced around
hesitant to give up their managerial roles overseeing him.the
A D) According to one national survey, the average
70 single blue, fluorescent tube was burning on the wall. The characters’ behavior during the gift giving
full-time US employee works about 1,700 hours
production
It was notof new
a bigvehicles. In order
room. Despite itsto retain these
bareness, the top mainly serves to
per year.
impression it gave was of austerity rather than A) emphasize the lavish value of the gift.
engineers,
poverty. theItcompany
made oneproposed
remember a creative 35 solution;
that poverty meant
displacement as well as lack, while austerity meant B) inflate the significance of the gesture.
job sharing. 35
75 being poor in a rooted way, within a tradition and C) convey indifference toward the gift.
As the demand
culture for flexible
of sparseness, whichworking options
transformed in the
even A) stress
D) NO CHANGE
the need for polite behavior.
lack, the paucity, into a kind of being. B) solution. Job
today’s marketplace has grown, job 36 sharing, an
C) solution job
arrangement in which one full-time job is split between 5
1 D) solution: job
two employees—has become more common. For Which choice provides the best evidence for the
According to the passage, the old man was standing answer to the previous question?
on the verandah because
36 A) Lines 43-44 (“It was . . . yoghurt”)
A) he was watching cars travel down the road.
A) Lines
B) NO CHANGE
44-46 (“they . . . unencountered”)
B) the two boys had reported the visitors would
B) Lines
C) sharing—
52-54 (“Oh no . . . all this”)
soon arrive.
C) Lines
D) sharing;
58-60 (“I have . . . stop me”)
C) he had heard what he believed to be the
visitors’ car. D) sharing:
D) he enjoyed listening to the quiet sounds of 6
the evening.
The description of Chhotomama and the son’s
interaction mainly serves to
2 A) show how the characters diverge in their
In the passage, the yoghurt and sweetmeats are approaches to cultural practices.
compared to a B) emphasize the characters’ complex relationship.
A) jewel. C) stress the characters’ misinterpretations of
B) cuisine. Indian history.

C) wedding gift. D) depict how the characters created gestures that


became routine.
D) generous donation.

April QAS 2017

Unauthorized copying or reuse of any part of this page is illegal. 29


3 CO NTI N U E
1
2 1
2
Question-and-Answer Service

Reading Test
employees, it provides an appealing alternative to other 37

...............................................................................................................................................................................................................................................................................................................
forms of part-time work, which might not offer Which choice best combines the sentences at the
65 M I NU TES, 5 2 QUESTIONSunderlined portion?
comparable health benefits, salary, or stability. For Levine
A) ladder, and they weren’t having
and Rocco, who wereTurncompensated
to Sectionat 180ofpercent of theirsheet to answer the questions in this section.
your answer
B) ladder; however, they didn’t have
full-time salaries and benefits, job sharing was a way of C) ladder without having
continuing to climb the career 37 ladder. They didn’t D) ladder while still not having

have to work the longEach


hours usuallyordemanded
passage of an below is followed by a number of questions. After reading
pair of passages
each passage or pair, choose the best answer to38each question based on what is stated or
employee in a high-level position.
implied in the passage or passages and in any accompanying graphics (such as a table or
Effective communication
graph). is crucial to the success of a A) NO CHANGE
B) there were steps taken by Levine and Rocco to
job-sharing arrangement. In determining how working ensure
time and responsibilities would be divided, 38 steps C) Levine and Rocco took steps to ensure
Questions 1-10 are based on the following old man’s listening ear, and to his wife’s ear, even
.......................................................................................................................................................................................................

werepassage.
taken by Levine and Rocco to ensure that the D) Levine
when and
the car wasRocco’s steps
relatively farensured
away and beyond their
This passage is adapted from Amit
range of vision. They had pondered over the sound,
arrangement provided fluidity and Chaudhuri,
consistencyA Strange
for the and 30 and finally, he had lit the lantern and shuffled out.
Sublime Address. ©1991 by Amit Chaudhuri. A ten-year-old
39 “I told her,” he said, referring to his wife. “I told her
employees whom
boy named they jointly
Sandeep managed.
travels with “It’s his
his mother, ouraunt
job to be
(Mamima), and his uncle (Chhotomama) to visit family in that
A) NO I heard the car, I knew it was the car, I told her
CHANGE
seamless,” they
Calcutta, noted. 39 Nevertheless, they each
India. you were coming.”
B) Once
To this end,
they were inside, Mamima gave the pot of
plannedTwo boysthree
to work were days
playing carrom
a week, ona the
with daysteps of a
of overlap 35 yoghurt and the pot of sweetmeats to the old
C) However,
small, painted shed which had the following words lady. “There was no need,” she said. “Oh really,” she
on Wednesdays;
on its wall inthey
large,also planned
black letters:toNATIONAL
talk on the phone D) Similarly,
said. “This is too much,” she insisted, with the air of
Line ASSOCIATION OF SPORTSMEN. A single
one who has just received the Kohinoor diamond as
5 table-tennis table inside the shed could be glimpsed
a birthday present. “Come, come, come,” said
through the window. The boys interrupted their 40 Chhotomama, with the air of someone who has just
game to give Chhotomama directions to the house given the Kohinoor diamond as a birthday present,
in a series of sporadic, enthusiastic gestures. Oh yes, and refuses to be overawed by his own generosity.
they knew the old couple. And yes, their son and “It’s nothing.” It was nothing, of course, only
10 daughter-in-law had arrived last night with their
Ganguram’s sweets and yoghurt, but they fussed and
first child. 45 fussed and created the illusion that it was something,
“Is it a girl or a boy?” asked Mamima, rolling something unique and untasted and unencountered.
down the window. The son and the daughter-in-law emerged shyly
“A girl,” said the boy. from the anteroom. They both stooped gently to
15 Mamima rolled up her window before the touch Chhotomama’s feet, and Sandeep’s aunt’s and
mosquitoes came in. The two boys vanished behind 50 his mother’s feet, a traditional greeting and a mark of
them. When they reached the house, they found that obeisance towards one’s elders.
the old man was waiting on the verandah with a “Oh no no no,” said Chhotomama, struggling to
lantern in his hand. Moths were shuddering round keep the son’s hand away from his feet. “There’s no
20 and round the lantern, though the old man was
need for all this.” This was half a token gesture
oblivious to them. He had come out because he had 55 towards modesty, and half towards the new,
heard the throbbing of the engine in the distance. “modern” India—Nehru’s secular India, free of ritual
The night had been silent except for the questioning and religion.
cry of an owl and the continual orchestral sound of
25 crickets in the bushes. The throbbing of the engine
had, therefore, travelled through the silence to the
April QAS 2017

Unauthorized copying or reuse of any part of this page is illegal. 2


30 CO NTI N U E
1
2 1
2
Question-and-Answer Service

“I havetonot
each evening met you
discuss for two
the day’s years,40Dada,”
work. The jobsaid the 40
3

...............................................................................................................................................................................................................................................................................................................
son, struggling to get his hands near Chhotomama’s
shared by“You
60 toes. Levine andnot
must Rocco
stopwas
me.”theThis
highest‑ranking
was half a token Which
As usedchoice
in linesbest
37 sets
and up
40,the information
“air” most nearlythat
means
gesture towards modesty, and half towards the old, follows in the next part of the sentence?
shared job at the company: when “you have to analyze A) atmosphere.
“traditional” India—Gandhi’s India of ceremony and A) absence.
NO CHANGE
B)
yourcustom.
day and share it with another brain, you show up
Sandeep, meanwhile, had come to the conclusion B) demeanor.
C) Levine and Rocco initially had doubts that the
the
65 next
that day ready to run,”
the grown-ups wereLevine
mad, said.
each Studies
after hishave
or her arrangement would be successful:
D) melody.
own fashion. Simple situations were turned into C) The job shared by Levine and Rocco requires
confirmed that job sharing can improve work quality by
complex, dramatic ones; not until then did everyone about 80 hours of work total per week:
feel important
encouraging and happy.
teamwork. Willsurvey
In a 2003 they never grow up?at
of employees 4 D) Both Levine and Rocco found that this
thought Sandeep irately. He glanced around him. A collaboration enhanced their job performance:
the
70 UK National
single Health Service,
blue, fluorescent for example,
tube was burning onmorethethan
wall. The characters’ behavior during the gift giving
It was not a big room. Despite its bareness, the mainly serves to
70 percent of job sharers felt that communicating with
impression it gave was of austerity rather than 41 A) emphasize the lavish value of the gift.
theirpoverty.
partnersItimproved
made onetheir
remember
ability that poverty meant
to understand and
displacement as well as lack, while austerity meant B) inflate
Which thebest
choice significance of the
sets up the maingesture.
discussion of the
execute
75 being their jobs.
poor in a rooted way, within a tradition and paragraph?
C) convey indifference toward the gift.
culture of sparseness, which transformed even the
41 Job sharing may present some challenges, D)
A) stress the need for polite behavior.
NO CHANGE
lack, the paucity, into a kind of being.
though. If job sharers have 42 discordant capabilities or B) Moreover, some workers have responsibilities
that are difficult to share.
are 5
1 unable to communicate effectively, the arrangement C) Still, only some positions are suitable for job
Which choice provides the best evidence for the
sharing.
mayAccording
not be successful. Job-sharing
to the passage, initiatives
the old man wasmay also
standing answer to the previous question?
on the verandah because D) Flexible work arrangements can reduce stress in
involve some extra cost for companies, since the salaries A) employees.
Lines 43-44 (“It was . . . yoghurt”)
A) he was watching cars travel down the road.
B) Lines 44-46 (“they . . . unencountered”)
B) the two boys had reported the visitors would
soon arrive. C) Lines 52-54 (“Oh no . . . all this”)
42
C) he had heard what he believed to be the D) Lines 58-60 (“I have . . . stop me”)
A) NO CHANGE
visitors’ car.
B) contrary
D) he enjoyed listening to the quiet sounds of 6 C) irreconcilable
the evening.
The description of Chhotomama and the son’s
D) mismatched
interaction mainly serves to
2 A) show how the characters diverge in their
In the passage, the yoghurt and sweetmeats are approaches to cultural practices.
compared to a B) emphasize the characters’ complex relationship.
A) jewel. C) stress the characters’ misinterpretations of
B) cuisine. Indian history.

C) wedding gift. D) depict how the characters created gestures that


became routine.
D) generous donation.

April QAS 2017

Unauthorized copying or reuse of any part of this page is illegal. 31


3 CO NTI N U E
1
2 1
2
Question-and-Answer Service

Reading Test

...........................................................................................................................................................................................................................................................................
and cost of benefits for two job-sharing employees are 43
usually higher than 43 being for a single employee. A) NO CHANGE
65 M I NU TES, 5 2 QUESTIONSB) that of
However, proponents contend that the investment is
C) those for
worthwhile because 44 they
Turn to enable
Sectioncompanies to retainsheet to answer the questions in this section.
1 of your answer
D) DELETE the underlined portion.
the most talented employees. As can be seen from Levine
and Rocco’s success in developing a best-selling 2011
44
sport-utility vehicle, job sharing provides an effective
Each passage or pair of passages below is followedA)
by aNO
number of questions. After reading
CHANGE
each passage or pair, choose the best answer to each question based on what is stated or
flexible working solution when undertaken by motivated B) those enable
implied in the passage or passages and in any accompanying graphics (such as a table or
employees and companies.graph). C) any of them enables
D) it enables

Questions 1-10 are based on the following old man’s listening ear, and to his wife’s ear, even
.......................................................................................................................................................................................................

passage. when the car was relatively far away and beyond their
This passage is adapted from Amit Chaudhuri, A Strange and
range of vision. They had pondered over the sound,
Sublime Address. ©1991 by Amit Chaudhuri. A ten-year-old 30 and finally, he had lit the lantern and shuffled out.
boy named Sandeep travels with his mother, his aunt “I told her,” he said, referring to his wife. “I told her
(Mamima), and his uncle (Chhotomama) to visit family in that I heard the car, I knew it was the car, I told her
Calcutta, India. you were coming.”
Once they were inside, Mamima gave the pot of
Two boys were playing carrom on the steps of a 35 yoghurt and the pot of sweetmeats to the old
small, painted shed which had the following words lady. “There was no need,” she said. “Oh really,” she
on its wall in large, black letters: NATIONAL said. “This is too much,” she insisted, with the air of
Line ASSOCIATION OF SPORTSMEN. A single
one who has just received the Kohinoor diamond as
5 table-tennis table inside the shed could be glimpsed
a birthday present. “Come, come, come,” said
through the window. The boys interrupted their 40 Chhotomama, with the air of someone who has just
game to give Chhotomama directions to the house given the Kohinoor diamond as a birthday present,
in a series of sporadic, enthusiastic gestures. Oh yes, and refuses to be overawed by his own generosity.
they knew the old couple. And yes, their son and “It’s nothing.” It was nothing, of course, only
10 daughter-in-law had arrived last night with their
Ganguram’s sweets and yoghurt, but they fussed and
first child. 45 fussed and created the illusion that it was something,
“Is it a girl or a boy?” asked Mamima, rolling something unique and untasted and unencountered.
down the window. The son and the daughter-in-law emerged shyly
“A girl,” said the boy. from the anteroom. They both stooped gently to
15 Mamima rolled up her window before the touch Chhotomama’s feet, and Sandeep’s aunt’s and
mosquitoes came in. The two boys vanished behind 50 his mother’s feet, a traditional greeting and a mark of
them. When they reached the house, they found that obeisance towards one’s elders.
the old man was waiting on the verandah with a “Oh no no no,” said Chhotomama, struggling to
lantern in his hand. Moths were shuddering round keep the son’s hand away from his feet. “There’s no
20 and round the lantern, though the old man was
...

need for all this.” This was half a token gesture


oblivious to them. He had come out because he had towards modesty, and half towards the new,
STOP
55
heard the throbbing of the engine in the distance. “modern” India—Nehru’s secular India, free of ritual
The night had been silent except for the questioning and religion.
cry of an owl and the continual orchestral sound of
25 If you in
crickets finish before
the bushes. time isofcalled,
The throbbing you may check your work on this section only.
the engine
had, therefore, travelled through the Dosilence
not toturn
the to any other section.
April QAS 2017

Unauthorized copying or reuse of any part of this page is illegal. 2


32 CO NTI N U E
Ali K
SECTION I I I PRACTICE SAT 2 1 29

Math Test- No Ca lculator


2 5 MI N U T E S , 20 Q U E S T I O N S March 2017
Turn to Section 3 of your answer sheet to answer the questions in this section.

D I RECTI O N S

F o r q uestions 1 -1 5, solve each problem, choose the best answer from the choices provided,
and fill in the corresponding circle on your answer sheet. For questions 1 6-20, solve the
problem and enter your answer in the grid on the answer sheet. Please refer to the directions
before q uestion 16 on how to enter your answers in the grid. You may use any availa ble space
in your test booklet for scratch work.

1 . The use of a calculator is not permitted.


2. All variables and expressions used represent real numbers u n less otherwise indicated.
3 . Figures provided in this test are d rawn to scale un less otherwise indicated.
4 . All figures lie i n a plane u nless otherwise indicated .
5 . Unless otherwise indicated, the domain of a given function f is the set of all real numbers x for which
f(x) is a real number.

G
AC=21t=7tr2r D·
A =lw �
A=lbhb
'� I
a
�x

Special Right Triangles

V=lwh V= -/wh
I
3

The number of degrees of arc in a circle is 360.


The number of radians of arc i n a circle is 27t.
The sum of the measures in degrees of the angles of a triangle is 1 80.

CONTI N U E

- 200 -
3
Ali K
1 4
A contractor rents a power saw for an initial fee of (2x – 5)(x2 – 3x + 4)
$15 and an additional fee of $12 per day. Which of the Which of the following is equivalent to the expression above?
following represents the total cost c, in dollars, to rent
A) 2x3 – 11x2 + 23x – 20
a power saw for d days?
B) 2x3 – x2 + 23x – 20
A) c  27d
C) 2x3 – 5x2 + 8x – 20
B) c  15(d  12)
D) x2 – x – 1
C) c  12d  15
D) c  15d  12

5
Two warehouse employees need to move microwave ovens
and televisions that weigh 45 pounds and 85 pounds,
2 respectively, using a freight elevator. The freight elevator can
A baker is making cakes. The recipe requires 3 eggs
hold a maximum of 2,400 pounds. Together, the employees
per cake, and the baker has 100 eggs before
and their loading equipment weigh 400 pounds. What is the
beginning the first cake. If the baker uses the eggs
maximum number of microwave ovens the two employees
only for making the cakes, which of the following
can take on the elevator if both employees, the loading
expressions represents the number of unused eggs
equipment, and 20 televisions are in the elevator?
after c cakes have been made?
A) 5
A) 100  3c
B) 6
B) 100  3c
C) 7
C) 3c - 100
D) 20
D) 100 c
3

6
A hotel has two types of rooms: superior rooms that cost $100
per night and basic rooms that cost $80 per night. On a given
night, the hotel received $4,200 from renting 46 rooms. How
3 many superior rooms were rented on thatnight?
y  2 A) 20
y B) 21
Which of the following ordered pairs (x, y) is a C) 26
solution to the system ofequations above? D) 30
A) (-1,-1)
B) (-1,1)
C) (1,-1)
D) (1,1)

- 201 -
3
Ali K
7 10
Which of the following is equivalent to  y for all x > 0
2

1
4y2
and all y > 0?

A)
y
2
B) 
2

Triangle ABC above is similar to triangle DEF (not 1

shown) where vertices A, B, and C correspond to


y2

vertices D, E, and F, respectively. If DE = 10, what is C)  2 y


the area of triangle DEF?
A) 30
D)  6 y3
B) 60
C) 120
D) 180 11

8
3x – 6 = y
3y – 6 = x
If (x, y) is a solution to the system of equations above,
what is the value of x + y?
Which of the following could be the equation of the graph in
A) 3
the xy-planeabove?
B) 6
A) y = (x2 + 1)(3 – x)
C) 12
D) 18 B) y = (x2 + 1)(x – 3)

C) y = (x2 + 1)(x + 3)

D) y = (x2 - 3)(x + 1)
9
A local movie theater is holding a fund-raiser for the
South School. For each child ticket sold, the theater
will donate $2.25. For each adult ticket sold, the
theater will donate $3.50. Assuming the theater will
sell 150 child tickets, which inequality can be used to
determine the number of adult tickets, x, that will
need to sell in order to donate at least $1,000 to South
School?
A) 3.5x ≤ 662.5
B) 3.5x ≥ 662.5
C) 3.5x ≤ 1,337.5
D) 3.5x ≥ 1,337.5

- 202 -
3
Ali K
12 14
Which of the following is equivalent to 2 1 for x >0?

  2  1
The kinetic energy of an object k, in joules, can be

represented by the formula k = 1 mv2, where m is the


2 A) 3
  3
mass of the object, in kilograms, and v is the speed at
which the object is traveling, in meters per second. B) 2   3
  3
Which of the following correctly shows the speed of
C) 2  3
the object in terms of its kinetic energy and mass?
 2
 3  2

D) 3  4
A) v  2k   3  2
2

B) v  2m 15
k 3x3 + bx2 – 27x – 9b
In the polynomial above, b is constant. Which of the

C) v  2m following is a factor of the polynomial?


k A) x+3
D) v  2mk B) x+b
C) 3x3 + b
D) X2 + 9
13

16

In the figure above, P is a point on the ground 30 feet


from the base of a flagpole.The angle of elevation from
point P to the top of the flagpole is 40° , and the tangent
of 40° is approximately 0.84. Of the following, which is
the closest to the height of the flagpole? The graph of the linear function f is shown in the xy-plane
A) 21 feet above. What is the y- intercept of the line that is parallel to
B) 23 feet the graph of f and passes through thepoint (-4,0)?
C) 25 feet
D) 27 feet

- 203 -
Ali K

- 204 -
Ali K

- 231 -

- 205 -
4
Ali K
2

One brand of a commercial ice machine can produce a


maximum of 263 pounds of ice each day. If 1 cubic foot of ice
weights about 57.2 pounds, which of the following best
approximates the maximum number of cubic feet of ice the
machine can produce in one day?
A) 0.2
B) 4.6
C) 206
D) 320

(x + 1) - 2(x + 1) = 0
What is the solution x to the equation above?
A) -2
B) -1
C) 0
D) 1

4
At a large college, of the students working toward a degree in
1 English, 250 were selected at random and asked how many
Yin mowed m lawns each day for 8 days. In terms of
books on average they read each month.The results of this
m, what is the total number of lawns Yin mowed in
survey can be best generalized to which of the following
the 8 days?
populations?
A) 8m
A) All students at the same large college
B) 8
B) Any sample of 250 students at the same large college
m
C) All students working toward a degree in English at any
C) m8
college
D) 8 + m
D) All students working toward a degree in English at the
same large college

- 206 -
4
Ali K
5 7
A meteorologist records the daily high temperature, in
degrees Fahrenheit (°F), over the course of a week in
a city as shown below: Wechat kangkanglaoshi

In the figure above, the sides of a triangle are extended as


shown. What is the valueof x?
A) 80
Between which two consecutive days did the greatest
B) 70
increase in high temperature take place?
C) 60
A) Day 2 and day 3
D) 40
B) Day 4 and day 5
C) Day 5 and day 6
D) Day 6 and day 7
6
Over a two-year period, scientists searched for
neutrinos (subatomic particles) that are produced
outside the solar system. The energy, E, of those 8
In a soccer league, each team received 3 points for a win, 1
neutrinos is measured in units of 1012 electron volts point for a tie, and 0 points for a loss. The Blue Jersey team
(TeV). The table below shows the results of the has no losses and a total of 36 points after playing 18 games.
scientists’ investigation: Which system of equations could be used to solve for the
numberof wins and ties, where w is the number of wins and t
is the number ofties?
A) 3w + t = 36
w + t = 18
B) 3w + t = 18
w + t = 36
C) 3w + 3t = 36
w + t = 18
D) 3w + 3t = 12
Approximately what percentage of neutrinos detected
w + t = 18
had an energy of 90 TeV or less?
A) 18%
B) 25%
C) 64%
D) 75%

- 207 -
4
Ali K
9 11

An investor receives divided payments that are 5% of


a business’s monthly profits. The table below shows
the amount A, in dollars, the investor received and
the business profits p, in dollars, for each of four
months:

The table above shows the number of each of four different


types of music composed by Mozart and Haydn. If a concerto
composed by one of these composers is selectedat random,
what is the probability that Haydn was the composer?

Which of the following equations represents the A) 199


430
relationship between p and A, where k is a positive
B) 45
constant?
430
A) A = k p
C) 45
B) A = pk 199
C) A = p + k D) 45
D) A = kp 124

10 12
f (x) =  This week, Maxine can work a maximum of 30 hours and
needs to earn at least $425. Her job at a mall pays $12 per
g (x) = x2
hour, and her job at a college pays $15 per hour. If x represents
What is the value of f(g(-27))?
the number of hours worked at the mall and y represents the
A) 27
number of hours worked at the college, which of the
B) -27
following system of inequalities represents the situation?
C) 27 A) x + y ≤ 30
D) f(g(-27)) is not a real number x + y ≥ 425
B) x + y ≥ 30
12x + 15y ≤ 425
C) x + y ≤ 30
15x + 12y ≥ 425
D) x + y ≤ 30
12x + 15y ≥ 425

- 208 -
4
Ali K
13 15
Which of the following is equivalent to the What fraction of the adults surveyed gave a rating of 4 or 5 to

expression 1 – x + x2 – x3? the movie?

A) (x + 1)(x -1)2 A) 82
250
B) (x – 1)(x + 1)2
B) 99
C) (1 – x)(x + 1)2 250

D) (1 – x)(x2 + 1) C) 194
400

D) 99
194

Questions 14-16 refer to the following


information. 16
If the theater manager assumes the surveyed group is
representative of the first4,000 people to view the movie at
this theater, about how many of the 4,000 people would the
theater manager expect to rate the movie a 2 or lower?
A) 380
B) 760
On the first night of each movie’s release, the C) 1,064
manager of a movie theater asks the people D) 3,240

who saw the movie to rate it on a scale of 1


(worst) to 5 (best). The table above
summarizes the responses of all 400 viewers
of one particular movie.
14
How does the median rating of the adults who saw
17
the movie compare to themedian rating of the If a is a constant and a < 0, how many solutions does the
children who saw the movie?
equation   a  a have?
A) The median ratings are the same
A) No solution
B) The median rating of the adults is greater
B) 1 distinct solution
C) The median rating of the children is greater
C) 2 distinct solutions
D) The table does no give enough information to
D) Infinitely many solutions
compare themedians

- 209 -
4
Ali K
Questions 18 and 19 refer to the 20
A manufacturer packages vegetables in cans that are in the
following information.
shape of right cylinders with height 15 centimeters and
Men: tmen = - 0.1569x + 361.8 volume 750 cubic centimeters. If the manufacturer reduces
Women: twomen = - 0.2514x + 555.6 the volume of the cans to 600 cubic centimeters but keeps the
Since 1912, both men and women have competed area of the base the same, by how many centimeters does the
in 100-meter freestyle swimming races at the height of the can decrease?

Summer Olympic Games. The winning times for A) 3


B) 4
men and women from 1912 through 2012 can be
C) 5
modeled by the equations above, wher t represents
D) 6
the winning time, in seconds, in year x.

18 21
According to the predictions from the equations, if Vitrivius, a Roman architect of the first century BCE, set
the Summer Olympic Games had occurred in the year guidelines in his writings for the relationship between the
2006, which of the following is closest to the number height (rise) and tread (run) of stairs in a building. He wrote
ofseconds by which the men’s winning time was less that the rise should be between 9 and 10 inches, inclusive, per
than the women’s winningtime? step, and the run should be between 18 and 24 inches,
A) 2 seconds inclusive. Which of the following could NOT be the ratio of
B) 3 seconds rise to run for a set of stairs that follows this guideline?
C) 4 seconds
A) 1
D) 5 seconds 3

B) 5
12
C) 0.38
D) 0.49
19
According to the equations, which of the following
conclusions can be drawn concerning the winning
times for men and women from 1912 through 2012?
22
A) tmen decreases faster per year than twomen, and y = -(x – 1)(x – 2)(x + 3)
the value twomen - tmen is increasing as x In the xy-plane, which of the following could be a graph of
increases. the equation above?
B) tmen decreases faster per year than twomen, and
the value twomen - tmen is decreasing as x
increases.
C) twomen decreases faster per year than tmen, and
the value twomen - tmen is increasing as x
increases.
D) twomen decreases faster per year than tmen, and
the value twomen - tmen is decreasing as x
increases.

- 210 -
4
Ali K
Questions 23 and 24 refer to the 24
Which of the following is the best interpretation of the slope
following information.
of the graphh?
The payload package of a weather balloon is 5 feet
A) The ratio of the height of the payload package, in feet
above the ground before the balloon is launched above the ground, to the distance from the weather station,
from a weather station. After launch, the balloon in feet
rises for 30 minutes. The height of the payload B) The ratio of the height of the payload package, in feet
package is estimated to increase by 10 feet every above the ground, to the height of 5 feet
15 seconds. The function h gives the height h(x), in C) The ratio of the number of feet the payload package has
risen since launch to the number of seconds since the
feet, of the payload package above the ground in
weather balloon was launched
terms of the time x, in seconds, after the weather
D) The angle that the path of the balloon makes with the
balloon is launched.
ground
23
Which of the following represents h(x)?

A) h(x) = x + 5 25
3 1
B) h(x) = 2  2
  y  14
3
3
x + 2y = − 4
C) h(x) = 2   5
3 If (x,y) is the solution to the system of equations shown
above, what is the value of x+ y?
D) h(x) = 5  2 
A) -4
B) -2
C) 2
D) 4

26

The table above shows tidal predictions for Herring Cove.


According to the table, what is the difference between the
median predicted high tide and the median predicted low tide
over the 7-day period?
A) 23.6 feet
B) 23.9 feet
C) 24.2 feet
D) 24.9 feet

- 211 -
4
Ali K
27 30
x2 + y = 4x − 3
y = 1 − 2x
If (x1, y1) and (x2, y2) are two distinct solutions of
the system of equations above, what is the value of
|x1 – x2|?

A) 2
D) 6
C) 20

D) 44
In the scatterplot above, each point represents the quantitiy n
28
of a product sold bya business at price p, in dollars, for one of
9x2 + bx + 49 = 0
30 days. Which of the following equations best models the
In the equation above, b is a constant. If the equation
relationship between price and quantity sold?
has more than one real solution, which of the
A) n = 50p +200
following could be the value of b?
B) n = −50p +200
A) 7
C) n = −50p +300
B) 21
D) p = 1
C) -42 2
50 n
D) -3

31
At Mount Rushmore National Memorial, four sculptures of
29 United States presidents’ heads are carved into a granite
In a scale drawing of a rectangular living room floor,
mountain. The ratio of a sculpture’s head length to the actual
the width of the floor is 2 inches and the length is 4
president’s head length is 40 feet to 6 icnhes. What is the
inches. If the width of the actual living room floor is
sculpture’s head length, in feet, for a president with a head
w feet,which of the following function A could
length of 9inches?
represent the area, in square feet, of the actual living
room floor?
2
A) A(w)= w
8

B) A(w) =2w2

C) A(w)= 4w2
32
D) A(w) =8w2 Miguel is buying prizes for a raffle. He will buy one prize
that costs $50 and four prizes that cost $20 each.The
remainder of the prizes will cost $10 each. If Miguel can
spend no more than $300 on all the prizes, what is the largest
total number of prizes he can buy for the raffle? (Assume
there is no sales tax.) Wechat kangkanglaoshi

- 212 -
Ali K

- 212 -

- 213 -
Ali K

- 214 -
Ali K

)?

- 215 -
Ali K

March 2017 Ans. Key

Section 3 Section 4
1 C 1 A 21 A
2 B 2 B 22 B
3 D 3 B 23 C
4 A 4 D 24 C
5 B 5 D 25 C
6 C 6 C 26 B
7 C 7 A 27 C
8 B 8 A 28 C
9 B 9 D 29 B
10 A 10 A 30 B
11 B 11 D 31 21
12 A 12 D 32 22
13 C 13 D 33 16
14 D 14 C 34 1312
15 A 15 B 35 5
16 6 16 B 36 1.44
17 1.8 17 A 37 9
18 0 18 C 38 13
19 14 19 D
20 0.6 20 A

- 217 -

You might also like